You are on page 1of 322

All this data is obtained from links provided in the post by Ahmad Hassan Bhai.

https://web.facebook.com/groups/979966878809240/permalink/1009057789233482/

Surgery OSPE
Table of Contents

1. Books to go through
2. Examinations
3. OSPE format
4. Important surgery points
5. Instruments
6. Radiology
7. Past OSPEs
8. Observed stations
Books to go through
• Dogar and Bailey: like you did in theory.
• GIT is of extreme importance and
• so is endo
• and breast for this
• and sometimes there are patients of parotid tumors too
• but urology and cardiothoracic are the least tested in this since long cases nahin hotay inkay usually in gen surgery
ward
• plus you need to go through burns
• arteries, veins, lymphatics
• skin
• trauma (ATLS) too since they are the most tested portions in ospe, Ye Chapters tu you cannot afford to miss a thing,
baki ki read would do
• For Written form of examinations in short and long cases get your copy of the book Wardmate and follow the links
below to know how to perform these examinations
• Zahid mehmood is required for examinations of all other things not in wardmate but those things aren’t usually
tested (see the viva questions of everything if possible from zahid mehmood)
Examinations:
1) Surgery Examinations by Me
https://m.facebook.com/notes/wardmate-examination-guide-plus-ospe-related-suff-for-mbbs/surgery-examinations-by-
me/471427466565858/

2) Surgery Examinations By S.Das


https://m.facebook.com/notes/wardmate-examination-guide-plus-ospe-related-suff-for-mbbs/surgery-examinations-by-s-
das/471413529900585/
Ospe Format
https://web.facebook.com/ahmad.hassan.7161/media_set?set=a.10206250041370355.1073742133.1845046617&type=3
Format of OSPE
MBBS Final Professional

SURGERY
'M
~~ ~
.· '

Helpers
OSPE Total Marks 55
Total Stations 13 (02 Rest Stations)
05 Marks at Each Station
05 Minutes at Each Sta tion

• St atic Stations 09
OS General Surgery
01 each of the four sub-specialties
(Anaesthesia, Urology, Neurosurgery, Orthopaedics)

• Interactive / 02
Observed Stations General Surgery and Trauma Only

Short Cases Total Marks 100


At least 03

Long Case Total Marks 70


Important Surgery points
for OSPE
https://web.facebook.com/ahmad.hassan.7161/media_set?set=a.10204604422310907.1073742007.1845046617&type=3
Important Signs in Surgery Examination

s/no Disease of Signs


1 Gall bladder Murphy's sign
Boas sign
2 appendix Pointing's sign
Cough sign
Rovsing' s sign
Psoas sign
Obturator sign
3 Hernia Carnet's sign
(paraumbilical,
lncisional,
Epigastric)
4 inguinoscrotal Ring occlusion test
5 thyroid Stellwag' s sign
Lid lag (Von Graefe's sign)
Joffroy' s sign
Moebius sign
6 Varicose veins Schwarts test
Tourniquet test
Perthes test
Trendelenburg test
7 pancreas Grey turner sign
Cullen sign
IMPORTANT SIGNS & TESTS IN
SURGERY
BY Maryam Malik
Rawalpindi Medical College
Allen's test
• The hand is elevated and the
patient/person is asked to make a fist for
ALLEN'STEST •
about 30 seconds.
Pressure is applied over the ulnar and the
radial arteries so as to occlude both of
them.
• Still elevated, the hand is then opened. It
should appear blanched (pallor can be
Pallor produced observed at the finger nails).
by clenching • Ulnar pressure is released and the colour
should return in 5 to 15 seconds.[1]
• The Allen•s test is testing for abnormal
circulation. If color returns as described
above, the Allen•s test is considered to be
Radial artery "Positive". If color fails to return, the test is
Ulnar artery occluded occluded considered "Negative" and the ulnar
artery supply to the hand is not
sufficient.[1] Theradial artery therefore
cannot be safely pricked/cannulated.
Ballance's sign
• Constant dullness to percussion in the left
flank and shifting dullness to percussion in the
right flank seen with splenic rupture/hematoma
• a dull percussion resonance sound heard on the ri
ght flank of a patient lying in the left decubituspo
sition, an indication of a ruptured spleen. The sou
nd is caused by an accumulation of liquid blood o
n the right side and coagulated blood on the left.
Battle's sign
• Also called mastoid ecchymosis, is an
indication of fracture of middle cranial fossa of
the skull, and may suggest underlying brain
trauma.

-:;; (

I

j
I Battle's sign
Beck's triad
Cardiac tamponade
- Beck' s triad distended
neck veins

hypotension
muffled heart
tones
Bergman's triad
• Seen with fat emboli syndrome-
• 1. Mental status changes
• 2. Petechiae {often in axilla/thorax)
• 3. Dyspnea
Blumer's shelf
• Blumer's shelf is a finding felt in rectal
examination that indicates that a tumor has
metastasized to the Pouch of Douglas.
Boas' sign
• Boas's sign is hyperaesthesia {increased or
altered sensitivity} below the right scapula can
be a symptom in acute cholecystitis
Borchardt's triad
• Seen with gastric volvulus-
• 1. Emesis followed by retching
• 2. Epigastric distention
• 3. Failure to pass an NGT
Carcinoid triad
• 1. Flushing
• 2. Diarrhea
• 3. Right-sided heart failure

• Seen with 5-HT syndrome


Carcinoid heart disease is characterized by right-sided
valvular fibrosis (increased collagen) leading to tricuspid
insufficiency and pulmonary valve stenosis (TIPS);
Left-sided valvular lesions are not seen due to presence of
monoamine oxidase (metabolizes serotonin) in the lung.
Charcot's triad
• Charcot's cholangitis triad is the combination
of jaundice; fever, usually with rigors; and
right upper quadrant abdominal pain

• Charcot's neurologic triad is the combination


of nystagmus, intention tremor, and scanning
or staccato speech. This triad is associated
with multiple sclerosis
Chvostek's sign
• Seen with hypocalcemia - twitching of facial
muscles upon tapping the facial nerve
Courvoisier's law
Courvoisier's Law

• "In tl1e presence of jaundice a palpable


gallbladder is u11likcly to be due to stone''

If tl1c obstruction was due to stone, tl1c tl1ick


walled gallbladder would probably not distend
Cushing's triad
• Cushing's triad (not to be confused with the Cushing reflex) is a sign
of increased intracranial pressure. It is the triad of:
• 1. Hypertension (progressively increasing systolic blood
pressure). Widening pulse pressure (an increase in the difference
between systolic and diastolic pressure over time)
• 2. Bradycardia
• 3. A change in respirations, often irregular and deep, such as
cheyne stokes
• Cushing's triad suggests a cerebral hemorrhage in the setting of
trauma or an space occupying lesion (e.g. brain tumor) that is
growing and a possible impending fatal herniation of the brain.
Cushing's triad is named after an American neurosurgeon Harvey
Williams Cushing (1869-1939}
Cushing reflex
• The ischemia activates the sympathetic nervous
system, causing an increase in the heart's output by
increasing heart rate and contractility along with
peripheral constriction of the blood vessels. This
accounts for the rise in blood pressure, ensuring blood
delivery to the brain. The increased blood pressure also
stimulates the baroreceptors (pressure sensitive
receptors) in the carotids, leading to an activation of
the parasympathetic nervous system, which slows
down the heart rate, causing the bradycardia.
• The Cushing reflex is usually seen in the terminal stages
of acute head injury.
Dance's sign

• Empty right lower quadrant found in patients


{usually children) with ileocecal
i ntussusception .

...,

S81!53g8'·
~mass - I -~
,.... .... ":·..."
n~~:in, ,.~, ~.,.\.

. 1".,,."··~·. ....
. .. . .. ...
,•,,) • I • • •

. ••.·"..,. .,
........•-.,,_J"..·o.""'
,..... ,, 4,,, ('.'•
.
.,......

. .... .'.
.,__... ... . , . . . ... \ I 0

'•. f.h ,. ~:;; •.


\; ( ,·~ ,, -4-,
Fothergill's sign
• Used to differentiate an intra-abdominal mass
from one in abdominal wall - if mass felt when
tension on musculature, then it is in wall (sign
+ve)
• (Tension on musculature: flexion of rectus
abdominis)
Cullen's sign
• It is named for Thomas Stephen
Cullen {1869-1953), gynecologist who first
described the sign in ruptured ectopic
pregnancy in 1916. also present in acute
pa ncreatitis.
• Seen with retroperitoneal hemorrhage- bluish
discoloration of periumbilical area due to
blood migrating around to anterior abdominal
wall through fascia I planes
Fox's sign
• Fox's sign is a clinical sign in which bruising is
seen over the inguinal ligament. It occurs in
patients with retroperitoneal bleeding, usually
due to acute haemorrhagic pancreatitis.
Grey Turner's sign
• Seen with retroperitoneal hemorrhage-
ecchymosis or discoloration of flank as result
of dissecting blood from retroperitoneum
GREY TURNER 1 SIGN CULLEN 2 SIGN FOX 3 SIGN

1. Named after British surgeon George Grey Turner(1877- 1951)

2. Named for Thomas Stephen Cullen (1869·19S3), Canadian gynecologist who first
described the sign in ruptured ectopic pregnancy in 1916

3.Named after George Henry Fox(l846-1937). Americ,ao dermatologist


Goodsall's rule
• Goodsall's rule is applied to the examination of anal fistula.
It states that
• if the external opening of a fistula lies posterior to a line
drawn from 9 o'clock to 3 o'clock then it tracks around the
anus laterally and opens into the midline posteriorly.
• if the external opening lies anterior to this line, then it
opens directly into the anal canal (Anal fistulae course in
straight path anteriorly and curved path posteriorly from
midline)

• It should be noted that application of this rule is often


unreliable in anterior fistulas and those with underlying
disease
Hamman's sign/crunch
• Crunching sound on auscultation of heart
resulting from emphysematous mediastinum;
1
seen with Boerhaave s syndrome (rupture of
esophagus), pneumomediastinum, etc.
Howship-Romberg sign
• Seen with obturator hernia - pain along inner
aspect of thigh as result of nerve compression
Homans' sign
• Seen with DVT - calf pain on forced
dorsiflexion of foot

I
HAMPTON'S HUMP

• Hampton hump refers to a


dome-shaped, pleurally-
based opacification in the
lung most commonly due
to pulmonary
embolism and lung
infarction
IRISH'S NODE

• lrish's node is an enlarged left axillary lymph


node. Commonly related to gastric cancer
Krukenberg tumor
• Metastatic tumor to ovary {classically from
gastric cancer)
Kehr's sign
• Seen with splenic rupture - severe left
shoulder pain {referred pain from
diaphragmatic irritation)
Kelly's sign
• Used to identify ureter during surgery- visible
peristalsis of ureter in response to squeezing
or retraction
Laplace's law
• Wall tension= pressure x radius {thus, colon
perforates preferentially at cecum because of
inc. radius and resultant inc. wall tension}
Leser trelat sign
• Acute onset of multiple seborrheic keratoses
{often on the back) associated with GI
adenocarimonas, lung, breast, and urinary
tract cancers
..• •
- '

''
Lhermitte's sign

• sometimes called the barber chair phenomenon,


is an electrical sensation that runs down the back
and into the limbs. In many patients, it is elicited
by bending the head forward.ill It can also be
evoked when a practitioner pounds on the
posterior cervical spine while the neck is flexed;
this is caused by involvement of the posterior
columns.
• cervical spine cord pathology- cervical
myelopathy, spinal cord tumour or multiple
sclerosis
McBurney's point
• 1/3 distance from anterior iliac spine to
umbilicus on line connecting the two
Meckel's diverticulum rule of 2s
• 2% of population 2% are symptomatic Occur
within "'2 feet of ileocecal valve
• Meckel's diverticulum is aka Ileal diverticulum
or diverticulum ilei
Mittelschmerz

• Lower quadrant pain due to ovulation


Murphy's sign
• Seen in cholecystitis - cessation of inspiration
while palpating under right costal margin;
cannot continue to inspire deeply because it
brings inflamed gallbladder under pressure
Obturator sign
• Seen in patients with appendicitis/pelvic
abscess - pain upon IR of leg with hip and
knee flexed
Pheochromocytoma symptoms triad
• 1. Palpitations
• 2. Headache
• 3. Episodic diaphoresis (Sweating)

• HTN is a very important feature of


pheochromocytoma.
Pheochromocytoma rule of 10s
1. 10% bilateral
2. 10% malignant
3. 10% in children
4. 10% extra-adrenal
5. 10% multiple
Psoas sign
• Seen with appendicitis and psoas
inflammation - pain elicited by extending hip
with knee in full extension
Raccoon eyes
• Seen with basilar skull fracture - bilateral black
eyes
Reynold's pentad
• Seen in suppurative cholangitis-
• 1.Fever
• 2. Jaundice
• 3. RUQ pain
• 4. Mental status changes
• 5. Shock/sepsis
• {Charcot's triad plus #4 and #5)
Rovsing's sign
• If palpation of the left lower quadrant of a
person's abdomen increases the pain felt in
the right lower quadrant, the patient is said to
have a positive Rovsing's sign and may have
appendicitis.
Saint's triad
• The concurrence of hiatal hernia, diverticulosis,
and cholelithiasis

Diverticulosis: formation of multiple


non-inflammed diverticula in colon, typically
asymptomatic.
Silk glove sign
• Indirect inguinal hernia sac in pediatric patient
• While the child is in a supine position, the
inguinal area should be palpated. When the cord
structures are rolled against the pubic bone, the
practitioner may feel layers of processus
vaginalis slipping over one another, as if made of
silk.
Sister Mary Joseph's sign/node
• Metastatic tumor to umbilical lymph nodes
Virchow's node
• Metastatic tumor to Left supraclavicular node
{usually due to gastric cancer)
Virchow's triad
• Risk factors for thrombosis-
• 1. Stasis
• 2. Abnormal endothelium
• 3. Hypercoagulability
Trousseau's sign= Seen with
hypocalcemia

Trousseau sign:
(very uncomfortable & painful)

• A blood pressure cuff is inflated to 20mm Hg above systolic blood


pressure level.
• arterial blood flow to the hand is occluded for 3 to 5 minutes.
• Carpopedal spasm:
• Flexion at the wrist
• Flexion at the MCP joints
• extension of the IP joints
• adduction thumbs/fingers

luor.'20ll Hypocalcemia 11
Trousseau's sign of malignancy
• It refers to the spontaneous, recurrent
formation of clots in different places
{migratory thrombophlebitis) in patients with
malignancies
Valentino's sign
• Right Lower Quadrant pain from perforated
peptic ulcer, due to pus draining into Right Lower
Quadrant. it can lead to false diagnosis of
appendicitis.
• It is named after Rudolph Valentino (American
film actor) who presented with right lower
quadrant pain and was diagnosed appendicitis.
which turned out to be perforated peptic ulcer. He
subsequently died from an infection (Peritonitis)
resulting from surgery attempting to repair the
perforation.
Westermark's • Seen with pulmonary
sign embolus-
• focal peripheral
hyperlucency secondary to
oligaemia (aka hypovolemia)
(decreased pulmonary
vascular markings on CXR)
• central pulmonaryvessels
may also be dilated
Whipple's triad
• Evidence for insulinoma-
• 1. Hypoglycemia
• 2. CNS and vasomotor symptoms (syncope,
diaphoresis)
• 3. Relief of symptoms with administration of
glucose
INVESTIGATION OF CHOICE(IOC) .......all have been asked as MCQs before
Coutesy: farooq asghar- SMC
1 )for acoustic neuroma------------------- ---- gadolinium DTPA enhanced MRI
2)nasopharyngeal angiofibroma----------CECT scan
3)posterior fossa tumours------------------MRI
4)D VT------------------ --- --- --venous USG
5)pulmonary embolism-----------------CT chest with contrast
6)imaging of seminal vesicle and ejaculatory ducts------vasography > transrectal USG
?)extraintestinal amoebiasis--------ELISA
8)avascular necrosis--------------MRI
9)interstitial lung disease-----HRCT scan
1 O)bronchiectasis----------HRCT scan
11 )osteoporosis---------DEXA scan(dual energy x-ray absorptiometry)
12)acute ureteric colic--------non contrast spiral CT scan abdomen
13)renal TB early stage-----lVP
late stage------CTscan
14 )localization of pheochromocytoma------MRI
15)acute dissection of aorta-----transoesophageal ECHO
16)chronic dissection of aorta(in stable patients)----MRI
1 ?)congenital hypertrophic pyloric stenosis------USGI
18)extrahepatic biliary atresia-------peroperative cholangiogram
19)discrete swelling(solitary nodule) of thyroid-----FNAC
20)acute Subarachnoid haemorrhage(SAH)-----noncontrastCT HEAD
INVESTIGATION OF CHOICE:
• Single Bone Metastasis- CT
• Multiple Bone Metastasis- Bone scan
• Spine Metastasis- MRI
• Avascularnecrosis- MRI
•Bone Density/Osteoporosis-DEXA (Dual energy x ray absorptiometry)
•Aneurysm/AV Fistula-Angiography
•Dissecting Aneurysm (Stable) - MRI (Unstable )-Trans oesophageal USG
• Pericardia! Effusion-Echocardiography
•Lobulated pericardia) effusion-MRI > CT
• Minimum Pericardia! Effusion-Echocardiography
• Ventricular Function-Echocardiography
•Radiotherapy/Chemotherapy induced cardiotoxicity-Endomyocardial Biopsy
•Pulmonary Embolism- CECT> Pulmonary Angiography> V/Q Scan
•Interstitial lung disease(Sarcoidosis)-HRCT
•Bronchiectasis-HRCT scan
• Solitary Pulmonary Nodule- High resolution CT (HRCT)
•Posterior Mediastinal Tumor- MRI
•Pancoast Tumor (Superior Sulcus Tumor)- MRI
• Minimum Ascites/Pericardial effusion/Pleural effusion- USG
• Traumatic Paraplegia-MRI
•Posterior Cranial Fossa- MRI

• Acute Haemorrhage- CT
• Chronic Haemorrhage- MRI
• lntracranial Space Occupying Lesion- MRI
• Primary brain tumour- contrast MRI (Gold standard however remains to be
biopsy)
• Metastatic brain tumor- (Gadolinium) contrast enhanced MRI
• Temporal Bone-CT
• SAH Diagnosis- unenhanced CT
• SAH aetiology- 4 vessel MR Angiography> CT Angiography> DSA
• Nasopharyngeal angiofibroma-CECT scan
• Acoustic neuroma- Gadolinium DTPA enhanced MRI
• Obstetrics- USG
• Calcifications- CT
• Blunt abdominal Trauma- CT
• Acute Pancreatitis- CT
• GERD- pH manometer> endoscopy
• Dysphagia- Endoscopy
• Congenital hypertrophic pyloric stenosis- USG
• Extrahepatic biliary atresia- perioperative cholangiogram
• Obstructive Jaundice/GB Stones- USG
• Diverticulosis - barium enema
• Diverticulitis - CT scan
• Renal TB (early) - IVP (Late)- CT
• Diverticulitis - CT scan
• Renal TB (early) - IVP (Late)- CT
• Posterior Urethral Valve- MCU
• Ureteric stone- non contrast CT
• Renal Artery Stenosis- Percutaneous Angiography
• Extraintestinal Amoebiasis- ELISA
• Discrete swelling(solitary nodule) of thyroid- FNAC
SECTION ENDED
Surgery instruments by Dr.
Maryam Malik RMC
https://web.facebook.com/pg/wardmate/photos/?tab=album&album_id=538398286535442
M 1313S

By Maryam Malik-RMC

Courtesy Dc_Macyam_MaJik_BMC;___________
2 M 1313S

ra1

Courtesy Dc_Macyam_MaJik_BMC;___________
3
M1313S

Courtesy Dc_Macyam_MaJik_BMC;___ _ _ _ _ _ _ __
4
M1313S

Courtesy Dc_Macyam_MaJik_BMC;___ _ _ _ _ _ _ __
5
M1313S

Courtesy Dc_Macyam_MaJ ik_BMC;___ _ _ _ _ _ _ ____


6
M1313S

Courtesy Dc_Macyam_MaJ ik_BMC;.___ _ _ _ _ _ _ ____


7
M1313S


I

Courtesy Dc_Macyam_MaJ ik_BMC;___ _ _ _ _ _ _ ____


M1313S

Courtesy Dc_Macyam_MaJik_BMC;___ _ _ _ _ _ _ __

9
M1313S

0 e OC O I O 0
es·ar in's

Courtesy Dc_Macyam_MaJik_BMC;___ _ _ _ _ _ __
10
M1313S

eavers re rac

Courtesy Dc_Macyam_MaJik_BMC;___ _ _ _ _ _ _ __
11
M1313S

Courtesy Dc_Macyam_MaJ ik_BMC;.___ _ _ _ _ _ _ ____



12
1
I an e re rac or an e n ec s MOOS

Courtesy Dc_Macyam_MaJik_BMC;___ _ _ _ _ _ _ __
13
M1313S

Courtesy Dc_Macyam_MaJik_BMC;___ _ _ _ _ _ _ __
14
M1313S

oun re

Courtesy Dc_Macyam_MaJik_BMC;___ _ _ _ _ _ _ __
M1313S

Courtesy Dc_Macyam_MaJik_BMC;___ _ _ _ _ _ _ __
16
M1313S

Courtesy Dc_Macyam_MaJik_B ___


17
M1313S

~ R2H1ii'
lcr~Tlil!I!

Courtesy Dc_Macyam_MaJik_BMC;___ _ _ _ _ _ _ __
18
M1313S

Courtesy Dc_Macyam_MaJik_BMC;___ _ _ _ _ _ _ __
19

_ _ _ _ _____
_ _ _ _ _ _ _ ___
8MC_
. __ _ _
_aryarn Malik
rtes y Dr_M
20 M1313S

neur s a nee

Courtesy Dc_Macyam_MaJik_BMC;___ _ _ _ _ _ _ __
21
M1313S

orce

Courtesy Dc_Macyam_MaJ ik_BMC;___ _ _ _ _ _ _ ____


22 M1313S

Courtesy Dc_Macyam_MaJ ik_BMC;.___ _ _ _ _ _ _ ____


23 M1313S

Courtesy Dc_Macyam_MaJik_BMC;___ _ _ _ _ _ _ __
24
M1313S

Courtesy Dc_Macyam_MaJ ik_BMC;___ _ _ _ _ _ _ ____


25 M1313S
26 M1313S

oc ers orce

Courtesy Dc_Macyam_MaJik_BMC;___ _ _ _ _ _ _ __
27 M1313S

Courtesy Dc_Macyam_MaJ ik_BMC;___ _ _ _ _ _ _ ____


28 M1313S

Courtesy Dc_Macyam_MaJ ik_BMC;___ _ _ _ _ _ _ _ __


29 M1313S

Courtesy Dc_Macyam_MaJ ik_BMC;.___ _ _ _ ____


30
M1313S

orce

Courtesy Dc_Mac_Y-arn Mal ik_BM_C' - - - - - - - - - - - - - -


31
M1313S

• •

Courtesy Dc_Macyam_MaJik_BMC;___ _ _ _ _ _ _ __
32
ee e
e r e s s

a- I&
-
on
33
M1313S

Or Matet~

Courtesy Dc_Macyam_MaJi

..
34
M1313S

ro ene

Courtesy Dc_Macyam_MaJ ik_BMC;.___ _ _ _ _ _ _ ____


35
M1313S

Courtesy Dc_Macyam_MaJik_BMC;___ _ _ _ _ _ _ __
36
M1313S

Courtesy Dc_Macyam_MaJik_BMC;___ _ _ _ _ _ _ __
37 M1313S

Courtesy Dc_Macyam_MaJik_BMC;___ _ _ _ _ _ __
38 M1313S


I
39
M1313S

Courtesy Dc_Macyam_MaJik_BMC;___ _ _ _ _ _ _ __
M1313S

Courtesy Dc_Macyam_MaJik_BMC;___ _ _ _ _ _ _ __
41
M1313S

Courtesy Dc_Macyam_MaJ ik_BMC;.___ _ _ _ _ _ _ ____


42
M1313S

ca
43
M1313S

e esco e

Courtesy Dc_Macyam_MaJ ik_BMC;.___ _ _ _ _ _ _ ____


44 M1313S

v canu a

18G
20G
- .
l- . :

24G-

Courtesy Dc_Macyam_MaJ ik_BMC;.___ _ _ _ _ _ _ ____


SECTION ENDED
Surgery Radiology by Dr.
Maryam Malik RMC
https://web.facebook.com/pg/wardmate/photos/?tab=album&album_id=538396433202294
1
M1313S

BY MARYAM MALIK
Rawalpindi Medical College

Courtesy Dc_Macyam_MaJik_BMC;___ _ _ _ _ _ __
2
M1313S

• Any part of the bowel may be visible if it contains


gas air within the lumen. Gas air is of low density
and forms a natural contrast against surrounding
denser soft tissues. It is often difficult to
differentiate between normal small and large
bowel, but this often becomes easier when the
bowel is abnormally distended.
• The upper limit of normal diameter of the bowel
is generally accepted as 3cm for the small bowel,
6cm for the colon and 9cm for the caecum
proximal colon 3 6 9 rule . Sigmoid= 5cm

Courtesy Dc_Macyam_MaJik_BMC;___ _ _ _ _ _ __
M1313S

Courtesy Dc_Macya al i
M1313S

Courtesy Dc_Macy
M1313S

Courtesy Dc_Macyam
6
M1313S

Courtesy Dc_Macyam_MaJik_BMC;___
M1313S

• Intra-abdominal inflammation, such as with


pancreatitis, can lead to a localized ileus. This
may appear as a single loop of dilated bowel
known as a 'sentinel loop.'

Courtesy Dc_Macyam_MaJik_BMC;___ _ _ _ _ _ __
M1313S

Courtesy Dc_Macyam_MaJi
9
M1313S

• The Rigler sign, also known as the double wall


sign, is seen on an x-ray of the abdomen when air
is present on both sides of the intestine, i.e. when
there is air on both the luminal and peritoneal
side of the bowel wall.
• Pneumoperitoneum may be a result of
perforation or, from recent instrumentation or
surgery. A false double wall sign can result from
two loops of bowel being in contact with one
another.

Courtesy Dc_Macyam_MaJik_BMC;___ _ _ _ _ _ _ __
M1313S

ler
+

Courtesy DcMacya1·n Malik__. . .


• I ... ... t \YA'(l.
12
M1313S

• Sigmoid volvulus is a cause of large bowel


o struction and occurs when the sigmoid
colon twists on the sigmoid mesocolon .
• COFFEE BEAN SIGN on X-ray

Courtesy Dc_Macyam_MaJik_BMC;___ _ _ _ _ _ __
-

, }l J
1
I

MatC? G-,

.. ' .
.

' '
' .

' 1 ·'
t•
'.

. ··~


rlesy De
II version doesn't J)Ut this marl<.
14
M1313S

• Caecal volvulus describes torsion of


the caecum around its own mesentery which
often results in obstruction . If unrecognised
can result in bowel perforation and faecal
peritonitis.

Courtesy Dc_Macyam_MaJik_BMC;___ _ _ _ _ _ __
M1313S

Courtesy D
16
M1313S

• may show a dilated pharyngeal pouch


• the presence of air in the stomach and bowel in
the setting of oesophageal atresia implies that
there is a distal fistula.
• if an oesophago-gastric feeding tube insertion
has been attempted this may show the tube blind
looping and turning back at the upper thoracic
part of the oesophagus or heading into the
trachea and or bronchial tree.

Courtesy Dc_Macyam_MaJik_BMC;___ _ _ _ _ _ __
M1313S

sy D

18
M1313S

• Oesophageal atresia without fistula. The


Replogle tube arrow is coiled in the dilated
blind ending oesophageal pouch. AXR
demonstrates absence of bowel gas indicating
that there is no distal tracheoesophageal
fistula

Courtesy Dc_Macyam_MaJik_BMC;___ _ _ _ _ _ __
19
M1313S

Courtesy Dc_Ma,_.. ., .
d with trial version of Visual Watermark.Full versi
20 M1313S

ue ena

Court - MaJik_BMC;___ __
21
M1313S
22
c a asia : ir 's ea a ea rance onMOOS
23 • a
ar1u ene
Ml3l3S

• •
1n a ea I

Lead pipe appearance of colon is the classical


barium enema finding in chroniculcerative colitis.
There is complete loss of haustral markings in the
diseased section of colon, and the organ appears
smooth walled and cylindrical.

Courtesy 0[
24
M1313S

Barium X4ay

Courtesy Dc_M acyam_M aJ i __BMC;.___ ___~ - - = = = - - = - - -- - . . . .


25 M1313S


I
26
M1313S


as r1c u cer

Gast.tic
urc (GU 1
p oric n u""
27 M1313S


as r1c u cer
28 M1313S

The shoulder sign, the impression of the


hypertrophied pyloric muscle on the distended
gastric antrum

Courtesy Dc_Macyam_MaJ ik_BM_C;...____ _ _ _ _ _ _ _ _ __


29 M1313S

Magnetic Resonance
Chol a ngiopa ncreatogra phy

Courtesy Dc_Macyam_MaJik_BMC;___ _ _ _ _ _ _ __
30
M1313S

"Normal hepatic ductal anatomy". Coronal otDlig la~_Ml~~-


reformat image reveals the confluence circle between
the right posterior duct RPD and the right anterior
duct RAD , originating the right hepatic duct RHD .
Note that the RPD has a more horizontal route while
the RAD is more vertical. By its turn the RHD joins the
left hepatic duct LHD , originating the common hepatic
duct. The LHD results from the confluence of the ducts
of the left hepatic lobe segments, here only
represented by segments II 52 and Ill 53 . Cystic duct
CD , common bile duct CBD , main pancreatic duct
MPD , gallbladder GB .

Courtesy Dc_Macyam_MaJ ik_BMC;___ _ _ _ _ _ _ ____


G
M P l)

RHD

Cour . .-

_ Protected version Full version doesn't put this mark.


duct (MPD) crossing the whole pancreas and continuing as the duct of Wirsung (DW) at the
pancreatic head. At its distal portion the DW joins the common bile duct (CBD), araining
into the major papilla (pointing triangle). The duct of Santorini is not demonstrated
-

Courtesy
M1313S

•Jrn.. - I -.,,p in -ge t:'.ho u.•• ing the conon bile duct
11

Courtesy D -- . r·". e p ncre ,tic d.u ct ., - rr·o, . r ote the



34 •
s o In s one I n o eF ar o MBBS

Mate>~~

MRCP image shows a


dilated bile duct with a
dark stone (arrow) in its distal end.

Courtesy Dc_Macyam_MaJ ik_BMC;___ _ _ _ _ _ _ _ _ ____


35
M1313S

Endoscopic Retrograde

Courtesy Dc_Macyam_MaJik_BMC;___ _ _ _ _ _ _ __
M1313S

do co
p II P n duod n : '
Cou r~s91 B~~fYaMCf9f~ 1ILRM_C=---------------=---=-----------
37
These two fluorospot images taken during an ERCP.,~demonstrates s
stones in the common bile duct on the left radiogr;a ph, and cystic:
~-

Co
38 M1313S

ercu aneous

PTC (Percutaneous
Transhepatic Cholangiography) :
It is a radiologic technique
1.
INl'Cdlc used to visualize the
·n$CrtCd
anatomy of the biliary tract.

2. co,11rnon
Till:cdlc b1I<: duct
lnj .,et .
con ras
medtum
nto tho Dilated
bile ducts lncrahepatic
(from right side) bl a,y dues

/ •

L~t1tio /
Co~r~~Y Dc_Macyam_MaJik_
M1313S

.,. ,
. ,



,I •

Courte
Protected with trial version o . iJterm ark. Fu 11 version doesn't put this mark.
40 M1313S

T-tube cholangiograms are a fluroscopic study performed in the setting of hepatobiliary


disease.

41 • • M1313S

ero era 1ve c o an 10 ra ur1n


c o ec s ec o

Court
M1313S


I

Courte
43
M1313S

Courtesy Dc_Macyam_MaJ ik_BMC;___ _ _ _ _ _ _ _ __


M1313S

••
;_.
~

I witffi tria l version of Visual Watermark. Full version doesn't putt


45
M1313S


1n

A plain KUB showing a


large smooth oval
radio-opaque mass
suggesting a calcified bladder stone

Courtesy Dc_Macyam_MaJ ik_BMC;___ _ _ _ _ _ _ _ _ ____


46 M1313S
47
M1313S

Courtesy Dc_Macya
M1313S


ur1nar a

A cystogram showing marked diverticulation


with a large superior diverticulum.

Co
49
M1313S

e ro ra e re ro ra
ure

S-tricture

Tip of the penis

No,rmal Urethra
50
M1313S

a
PECTORALIS MUSCLE

Courte
51
M1313S

Courtesy Dc_Macyam_MaJ ik_BMC;___ _ _ _ _ _ _ _ _ ____


e -
M1313S
53 M1313S

Cour
54
M1313S


c av1cu ar

Courtesy Dc_Macyam_MaJik_BMC;___ _ _ _ _ _ _ __
M1313S

• •
1s oca 10n

Courtesy 0[___ Mali


M1313S

Co urte sy D c_M a,_•---::--r-


l with trial version of Visual Wate rm ark.Full version doesn't put thi
M1313S

• •
1s oca 10n

Courtesy Dc_Macyam_MaJik_BMC;___ _ _ _ _ _ _ __
58
e rac ure o 0 er en BS

u na

Courtesy 0[___ alik


M1313S

Courte

60 M1313S

ra s o 1n nee o e
rac ure

Courtes
61
M1313S


I

Courtesy D
62
us in oore ros esisoos

Courtesy Dc_Macyam_MaJik_BMC;___ _ _ _ _ _ __
63
M1313S

Courtesy Dc_Macyam_M
M1313S

'-
!t
·•~ ...._
f

---

Courtesy Dc_Macyam
rial version of Visu al Wate rmark.Full version doesr
65
M1313S

• •
nee
Courtesy Dc_Macyam_Ma
version of Visual Watermark.Full versio
67
M1313S

Courtesy Dc_Ma
• M1313S

I-

I e
• •
01n

Bimalleolar fracture and right ankle dislocation


on X-ray (anteroposterior). Both the end of the
fibula (1) and the tibia (2) are broken and the
malleolar fragments (arrow: medial malleolus,
arrowhead: lateral malleolus) are displaced.

Protected with trial version of Visual


69
rac ure o st

one

Preoperative radiographs showing the dorsal


dislocation at the MTP joint and fracture of the base
of the metatarsal bone.

Cour i~BMC~ - - - - - - - - - -
70
u i e e as as is i n os
71
M1313S

-
,__,~~'.•'} '-".,_,. , ._. •-•• ,_,, _ , --.v•-

StlJd~/ T1rne 9 50 09 A
r·.-1Rt~ 4 700~

Courtesy Dc_Macyam_MaJ ik_BMC;___ _ _ _ _ _ _ _ _ ____


72
M1313S

Courtesy Dc_Macyam_MaJik_BMC;___ _ _ _ _ _ _ __
73 M1313S

'-
-
---~-
- - • I j

-·- I
• I

Ir ~ , -- 1
I . I'

-' I

......
, ... r-
'
I

- -
1
I
" >
H I
I ..
I I I I
I
'
'

:.,·,.
. .....
:.1
.I ' ' I

r-- • . -
•~, 1_1 fl 1' 11.'1_1
T1 It .~•4 1:1
I :· ,-
. . ' • I

l. I r-, r.11 .. r....· ,i' L I r


Courtesy 0[_____ , I I .• _. - I
.' .

ected with . Full version doesn't ut thi


74
M1313S

race

Courtesy Dc_Macyam_MaJik_BMC;___ _ _ _ _ _ _ __
75
u i e ra c u res i • MOOS

Court
'•
I

•t I
'•
.a. •

'.
- ,
,- I ! •
; I f- I I


I
.'
' •• •••

)',,
, r' I
' ' 1 '.
-

I

' • I
-,_, •
\ • \

\

'
'
M1313S

Courtesy Dc_Macya
78
M1313S
CT angiogram of abdomen & lower limb showing normal vascul ature

Courtesy Dc_Macyam_MaJi
Courtesy Dc_Ma
This magnetic resonance angiogram ( ) of the lower extremities was M1313S
obtained by using the bolus-chase technique. A short-segment high-grade
stenosis is present in the middle of the left superficial femorial artery,:!--Note ~-
the collateral arterial supply.

" ,

' .~ I
'


,••' ,•

'

Protected with trial version of


81
M1313S

Magnetic resonance angiogram showing narrowing of


left popliteal artery Mate t~

Patlent with left-sided per1pheral arte1ial occlusive disease.


Font ·
Co u rte sy D c_M a,_.~,.-r,

82 • M1313S

1n u or 1n
ancreas

The arrow indicates


the superior mesenteric artery.
M1313S

ancreas

Courtesy Dc_Macyam_MaJik_BMC;___ _ _ _ _ _ _ __
84 M1313S


I

Courte
85 M1313S

Courtesy Dc_Macyam_MaJ ik_BMC;___ _ _ _ _ _ _ ____


86 M1313S

¥en

Caudate
lobe

Courtesy Dc_M
87 M1313c:

• Multiple hypodense lesions seen in the liver


_1ver with no significant contrast enhanc;ement.~_

Courtes
88 M1313S

•Oval, well defined


•Imperceptible or thin wall
•Water density

Courtesy Dc_Macyam_MaJ ik_BMC;___ _ _ _ _ _ _ _ _ ____



1ver arro
SECTION ENDED
Surgery OSPE
https://web.facebook.com/pg/wardmate/photos/?tab=album&album_id=537659343276003
1
- -
T a sk :
g,aph a n d a n ,w e , the
t, u m e n t / phop1o
es th e g iv e n ln s
C a .-eruny
g e
q xeasmtiin
u o n :
follo1vln
01
ott. oI
1 ldonofy thl, obJ
G iv e It s c o m m o n es t u s o 01
2. u. u d
e n c y •l lu n t1 o n 1 whn r• II r.en
3 . G ive n v o rm rr g OJ
c u r du11 lo fl r,rolo0<1fl<J ir .• 1
n tf o n s r o n o c
4 . W h a t c o m p ll c h v o fJ It !JC, p ln c« 1 ~ ,1t11
01
e d u ,e I~ nc -c,. • .,,y, wh111
s . I f Pro lo n g

KEY;
01
Cuffed e n d o tr a c heal rube 01
1.
In ju ry or after th , .., "..'idectomy etc.
.. 01
e sla . lflary
Jn gen e ra l a n a e sth ax wed b y stenosls
2. lt y In faclom
follo 01
Brea th in g d lf fl c u
J. necrosis
4. Tracheal pressure 01

Tracheotomy tu b
e
s.
For Candidate: 2
A 31 year old farm er was n Jn over by a trncto r
He WB:1 brul~ II " '"'( (h,· low• r
abd om en. His X• ray ls shown horc,

Task:

Carefully examine I hc given ra dlo(Jraph / photograph 11nd answer thc


fo llo1vlng questions:

l Whol bony lnju,y '""' uc, 1111 ,,111 (J1


2 , N.imt> 1\VO 1n,r1,11,1111 soil II 11<1 t11tu1h h1 10-, 11M li<,11( p;;,t ; i, th,1l rould have
occurred. I '.i
3 Name nn lmro1t,\11\ ,,1u11 1u 1ll,11,11,11r., n , " r,r llll!rn 1 !,
4 Nam~ nn ln1po1t,,n1 h\'""' J)·"·""' r,,m,,11< 1tlo11 lhltt ui11 o«.ur. 01
. -- - -

KEY:
01
l , Fracture pelvis & perlrochanterlc fracture of right femur. 15

2. rupture urethera & rectal tear d blood on fin ger stall after Pfl 1.5
3 Hematurea or blood on the tip of urethera an Ol

4 _ Haemorrhagic shock
vo 1e c c1v e1 y St ru c tu re d Pe rfo m -••- - vv •
an cc Ev alu a t io n ( OS PE )
3
s ta ti c station 1
M ar ks : OS
Tim e Al low ed : OS m lnu t e_s

F or Candid ate;,
A 39 y ea rs old -.mok,'1 l"e ,;, •n l cJ wi
th vnm111n,J 11, r ,.,,.,,, ftit•, X• r1,y wo~ 1.1i-en b
ho ur ilfl er tai..tng l\n 11un1 .-.u11,t111r1

Ta sk i

Ca re fu lly ex an ,ln th gl v n , ,_<llo gr •p h / ph ot og ra ph


nd an sw er th e
fo llo wi ng qu es ti on s:

l . \'/ ha t IS th e pr ob ab le dl.:ic;ino Is)


01
2. Na me an oth er im Porta nt 1nv t19
4t1 n wt k:h Sl"louio be dOne
01
3. If i t ,s a be nig n d,se.,se na ml " on
e op er t1v tre ., mc nt wt \ am be dO ne ,
1.S
4 . If ,t Is a ma llg na nt c.-i no,r ,on na me
on e op era tiv e tre atm el' \t wh ich sttoo
ld be
do ne ?

KEY:
- 1.S

1 . Pyloric. stenosls 01
2 Ga str os co py 01
3. Ga str oje jun os tom y l .S
4 . Radic al Ga ste re cto my 1.s
For Candidate: 4
A patient ha-. presented with r,al n tn right nonk and hllematurlt1 Hr,. t1bdOmlndl x-
rays are showr, h ere.

Task;

Carefully e xan,lno th e given rlldlograph / photograph and answer the


fo llowing questions:

01
i . What Is X•l..iy ') nnd wli,11 dol" II r.h,1wl (II
3 \Vh.it t~.)lment or u,11; '""111un1111'101110 Im 11 nof rJ1
01
-------------

X-ray 2 X-ray I

KEY:
1, Plain X-ray abdomen showrng radio opaque shodows In lumber area. 01
2 . IVU showing hydronephrosls In right kidney
01
02
3, Pyelollthotomy
01
4 Not llkely to be successfu l
F o r Ca n d id a te : 5
A 60 year ol d lady sllppe
d ond fell In her hi lthroom
ov er he r hand, th ,.•,<' are
ra y her 1<

Tas k :
Car e f ul ly ex an ,l n o th
o g lv o n x - rA y / p h ot ogr11ph an d a ns
w er th e
fo llo w in g q u e st io n s :

l \'\l ha l t-on, 1n1u,, ""~ t• l( ll fl l 'ln


01
2 \'>1h. ll dc to , m tt y .... lik 1•ly
h • 11, 11111 Ill I hn l lly / 01
3 \Vh.ll ,~ th e l rN tn, 1 nt '-'' th ht1t
l t11 1, I «;
4 ..H O\V IS l ht tr .ictu n tm

--- - - ----- -- --
•11 hl hl I I
1 c;
-
'

\ "
KEY ~
Ol
1• Colle's fr ac tu re
2 Dinne r fo rk de fo rm ity 01
·
3. Re du t tton by han d shake m et hod 1.s
1.s
4 _ con e's ptast:er:...:ca:.s_t ~ :,
Q ~ :: :: :l ? ~
M ark s: 0 5 6 Tim e Allowed: S m inut es

For Candidate;

You have received o patlcnl In the emergency who Is dyspnoc.:,c, his ~/H Is ;!§/ min,
pulse 120/mln, BP 180/100 mmHg and his race Is puffy. After lnltfaf management
you have received lollowlng AOG" rci,ort

pH "' 7.2S
Normal:
PCOi = 2S mmHg
• pH = 7.35 - 7.45
HC0:1 - l I mmol/1111~
• CO2= 35 - 45 mmHg
Ni! ' a I 3., n,Eq/t • HCO3-= 22-26 mmol/L
Cl = 9S mEQ/ l
• Na+ = 135-145 mmol/L
Task; • Cl- = 98-108 mmol/L

Caref ully read th a give n teen ar lo a 11d answ e r th e followi ng


questions:

1. What 1s your d1agnosls? Ol


2. What Is the probable unel~rlyrr,g cause? 01
3. What are the s~ps fn the management ot this patient? 03

KEY: -
1. Pure metabolJc acidosis with high 11n1onlc gz,p. 01
2. Acute renal tallure Ot
3. 03
Catheterlutlon to monitor urinary output
Send the urine for analysts
Administration of diuretics
Administration of 100% oxygen
Admfnlstrat1on of sodium bicarbonate sotutlon In adequate amount
For Ca ndidate: 7
This l 7 year old girl has presented with a swclltng of rron t of n<!r.l< :>tC?

T ask :

Carefull y exam in e t h e given photograph / patient nnd answer t he


following qu estions:

1. GlvP IWO likely dlt1gno•,1•'i OJ


2 If u,,s ghl undt:'l\lOt:~ t1,yl\1ltl11,\(llf\y ,1nd 1lrtv('flit)111i,ip1r lt>ry dl~t r~'; ti),. ..)ml•

J s

OC'C\lr DO~lCli't:f ,lllV~lv. I '>


4 . ThtS glr1 con1pl,11nt.. 111 l lh, \111)111 ,,t nurrthne!-~, Y1t1at OIIICf r;l(J • :M fO,.Jr , ,,. to
01
~hcit?
--·-------

KEY:
01
1 . Puberty goiter. Primary thyrotoxlcosls
1.5
ma tracheomalacla, pretracheal hematoma.
2. Laryngea I Oede , l.5
3. Hypothyroidism 01
4. Chvostek's slgn
m , .,. I,.. - ,. ... ,., .,_. .,, I

T a sk ;
8

C ar ef ul ly ex am in e th e gi
ve n ln st ru m on t an d an aw
or th e fo llo wing
qu es tions :

1. Id en tif y tt, ls 1ten, .


2. Name ONE co n,n,on u~o. 0I

3 Name TWO .-idvn nl ng "· \llhlrl 0I


) II hu• Oltl"r llijlln ,1 1111, •ih M I,)
0I
4 Name ONE Of 1\fl lh4'1 {lf11'U
llr 11\4''11
(J I
S ro r \11h.1l re liod 01 11n,,, , 1111
tl hi 1, "I'' 11,, Ol

KEY:
1. e·p1dural needle and cathet
er 01
2 • Epidural anesthesia Ql
d ces minimum metabolic dcfa
3. It pro u ngements at
• It Is not associated with spinal d he
hea ac
4. Epidural analgesia 01
s. Three days 01
I tm u l\ll n wnrl ' • IH l t UJ't'C•·

9
For Candidate:

/I 2 2 ycilr o ld lody hod rc•poo l c d vo m1t1nq nnd dl/11 1lu,11 ror 1111 I"' 1111y• Nnw :i1u Ii,~
prcs.:?ntod In th e on1erqcncy With obdomlnol d1r,l nnl'illlfl H11r pll1ln • r,,v 111lrlom•·n 1!1
shown 11<.' rc.

Task.
Carefully 0l(nn1lno th o glvon rndlogroph and nn• wor lho roll o w l nu
q u estions:

l VVh.:, t 1.:iOIOIO(l ll~ I rt n1ll1i 1)<i i,1,, 111 ihl• llrl l !IIG • lrifl fl 1
2 Wh() l 1!> lllC li kl•ly 1ll,l1AIII'''' ' ti l
3 Give 0 111) IX'''" ", fhhlhlll Whh h 1 Ii~ ly to I II' l'n l Ill 11,e ..,. ,,,, I <,t tltlli
p ,lllcn\ (11
4 G,vc ph) IC.)I llnlllnQ 1'11111, h I II•, ly I • '" 11rm;cnl CJ I

S HO\V would vou ll!."'11 lhl 1•'1lfnnt ' 01

KEY:
1. Mul tlple fluid leve ls 01

2 Para lytic lieus


01
3 Absence of passage of flatus and feces OJ
01
4 . Absence or bowel sounds.
01
s Drip and suction
,---:--,,.;:--,;;;-~;:""!l'.l'n;,--n;;~~
\
10
j •

' Matt?
.. 'I
• '

~ • •
,,.r" M BBS Final Pl'Qfil•elon1;1I
SURGERY ·
Ai, nu,nl ll1<•mlt1•.tl cn• 2008
ObJ~ct(vc (y St)1.tc~tJl'fJd Porfom ,nnco 18v•lu• t lon ( OS PE)· •

itatls: 5tatlon 2
,.....~.. . oq..
"-' • p l, , • Tim• ~1•ow~ OS minutes

'
• -


01.
• Ol •
2. Wbat-era presen~')fl!) ot peflductat ~157 l:
01 -~ •
~- What Is tr~unerit of pertductal ri,a~tls? •

01 ~
4,. Pertductal m~lils hat 90t mallk_. ~tlol'\
~ ,
Witt\? ~
••

.:..
.:J
~)
~
• ,. •
I -

..~.,; ~.
\~ ~

-
... •


. .,
• ,.
,.
,.
- •
,

-. ' •

.. ,
• • •
• ••
T c. ... ( ! •
·- '

,.
tr. . :.:.U cw tn g q ue stions : ..

;. ,\ ·ma !t:J ;>.:.t\.d,
tor ltf !IN
3 ~ ~~ TWO common lndlC:Mlo.
11

4 ~ ·1e 1' H ~E ! points of. 11ta


!"
r-care.
G ve 1~ REE cortll)ll~ I , .
. - ~.w .·..
,

Mat!?
Iv.JBS Fi n al Pl"Ofeostoi:tllll
SURGERY •
" A1,nuo1 B>u1mlr::111:1ori. 200S • . •
• !J~&cti vatv St r u ctured PerforinJJnc• SvaluBtlon ( JSPE)

·~
t

I Tlmo :..1towe'd1os ml,yutes


-

,• ~

.....
..,.,., l'-4
...... . .,,,,,.. . •

.!.. Tubt? tr.a~ ~ ff Pl~ ~tf, obtur" 1 wt: '!!er 'l:,;r ~ la•ned(. ~

w,m tapes. . • o:;s


~UKb~ KY
Annual E!1<11n,1n4'9ne 2008
O.iJj ~ctivc ly Structu~d Ptlrform11nco Ev11/u11tlon ( OSPI!)

. ....
Static statloo Z
,. ;irlc. i CS Thn• ~IIO'Nedt 01 mlnut09
) _:_:.i:, r .cand!dat f.1•
1,
Ta . l>,;~


.
Clt "":u!ly observo l:1,o provided • i>Klrn•n / phoia9f•J>h .Jn::1 •flSW•t
the :': ;1owln,i1 qu~5tiol'tat
,•
1.. ~ . ,~E' th~ spec,meii. o.s •• •
2~ \ ~!t rom;l'!ications Mav Jose Ir 1t Obltructs;
-e. ?et'/1\Jrete,rie junction .
...u.". . .
b.. Bll.U1 eters. O,i
0 •..J ..,..
c.. Sia~er neclc,
3. :;Jve rr~od or treatment If:
a. Stan~ Is i .5 cm anc:tJ'·. ll\ f\.l'IJI pi!IVlt ,
b. Stone rs wlthfr. 5 crn o.t al5t,I Ureter. O.~ ,
4. ! c,-me tnJ hype,j8cr1tlon of hormone which- may ~cs to ~urrence ot stone
02 "
14
;
• Mate
·'
MBBS fl.,,1 Ptofestlonal '
.. £ JR<if:!RY t

Annu• ' lhcnmlnatlon• 2008

- O bJoct /110 /)1 StrJ1cturt rJ ;,erlorm•nc• 61111/ulJtlon (OSP~)


11mo Alfowad: 05 mlnutc!l


• ..
:-i:::v.
• b ~ •,

. O~ te stone.
~ •
o.s

a, Hvctror....,httiSil, inft':ttk ·,1f ~ Cph:"OSH. 0.S



b. A."':.Jri..:. .o.s
c. Acu~ ....=tn;,ir-/ rt.t:)ntiOn. t•,S
• ~

i,.Pt.,"CU.-~ n ~ Y lf\d ~ o f ~- 0.5 .


.,_.
b. C-ystos:bpy and ~ ba_.
1r,<tnK.tlon. e.s
! J 02
·• PTH •
I
. ,.
.- C
:;..~
... in.s

-
,_ Tlmo A.llow1~ : l S mlr,utea
\I

-
• \
.a. k:
1 a ~ ful!y observ~ the p rQVad d p h otogre-m and .msweT the
-
_;,tami.ne.-r's qucstl:>ns .

• '
..

'

MBBS Flnal Profession al
Mate
SURGERY
, Ann uol l!lComlnutlons 2008
0..bJc ct l ilc /y S!ructurod Porforn1anco Evalua t io n ( OSPE)

I 11tera0tive staiion 1-0- I ...'->


•r 11,1 Q Allov1od1 OS mlnute5

-t( · V .• •

1

;,.:.:JJ.,w:;.y nd t crv cal ,;pine COl"lrl')i •


e 1:.-c!'::tung• •
C-'Cir\'.:.,l &i. on~ -.J hcm0t:i'111,ac conlrt •
:>- 'll:~ - iC,Cs;! c.ll oysfi.inctlOII~'
f -c:xpos1.;re. ir: ~ controHl!d en,lfror ment• 01
2.
~ ,I lnsp~ Qles! tor sc"')it .;I ~mphvsem.1, wouncts
a.1.: evt0ence c;f t--1.1 crj" ,I 01!!11t~tion.
eor.cioon of netl Vt.tis, symmetry of ch st, resptraio:--, e.~
,nd resplrato') r~te.
r.~ ~ on Jno ausc1.1 ltat1on. 01
~.usaJt:.C. t on fro m back, when PJtlent l09•rolted
3.
rneumothorax \Vlth c:ardlovascutar compromise due to red~ ven~us retum.
01
4. .
C'fagnosts ts cllnlcal llnc;I one should not watt tor X-rays. 0~
I

•· '-•- -'


i

rv e th a pr ov ld ud ob j ects / phot og ra ph and answ er


Ca r ~f'\,.!!y o!:::e
t he f ol lo1.vln:1 c:u cs tlo ns:

01
• 01
2 . G ,1e i:-d CJ lOf,S ror Ulic!,

n,n, on ml?dtcat con dition mo y leod to sud~~n obstruttJ.)I\ of an artery?


:: . Which co •
01

ul; tun:.u .ind blulsh lower limb? 01


4. Wh ich acu ll. condition Ju.,c.1~ to u 1,1..ilnt

ich obj1:c t Is used to cle ar thu ob str uc tion In vc$$~1:.? ' 01


5. Wh
• l
••
~
...\.
••
J
I. t
i
- '-"
,..,.18

f ,, I
I

,,,, .....
7
- .,
r J) •

!
I.
I•
! .i • ·--··

E_-,. C~ '.idid_, £.. i


-
----
t '. •

C c..- -3..t:,,t'
- t .... . } ~!Jscn:e the I rov1d ... •1 o!..!cc:l / p fl otogr'tlph nnd n n S\'Vcr

~me the object 05


2. ( 1e T'.'JO (!wO W•• C ,ndlC:l)t,ons If u ..od 10 SMll ••• 0 s. 0 S
3. c: ve T ,VO t, er .i...c ndlcat ons 0 s. o.s
£. ve THREE
E' .Name che cond
cl1r .. 1 aramc.:t rs for
.-
It$ removal efter surgery o.s. 0 s. 0 5
.J~ 11h,ch moy l<:£id to serlo~s resp ratory comp .c3tlon ccllo,•11.,g

- .... ._.
~" tJ"'' 01

-
...
• •

.M 8BS Fin 11 Pr ofessional


' SU RGERY
I

Annu.ll L xt n1lnt1 tlon11 2008
Objc c tfv~l y St r uc t u r od PQr (ortllOll CC l!v.i fu a t lon (OSPE)
.,
I. • statlc. statiodb.
f lrtJG A.llov, ed: OS m i nutes

KE :
- cs
1 . -.:, 1.11<--S ..tlc/ l"l" OQaSll'IC tub ,
1 . P1'k:rtc st~stsl trecll~~pl\Q9t ,1t sw~. C 5, O.S
3. P.. 1y T\',"O ttt~•a~e.:t c lndlcbU ,r.. l!:,.t'1•;;,,,,. ~ •, 0 .5, 0.5
4. ~ ' Y THR!:~ d ,. ,z:1 p~ramct ~ tor re 11.. ..1 wrgey. 0,5, 0.5, 0.5

---~'..-~
-· ....
,c.,,,,, ' -' ,I ... 01 '

"


I

• ••
20
For Candidate;
This Is an X-Ray of o 35 yeor old rcmalc who presented wltli upper obe1orn1na1 pain
and hos a history of chronic hcodacho

T ask ;
Car e fully oxomln tho gl v n rndlogroph/ photogrn ph and the answer

tho f ollowing qu • t lo1111

1 \~hnl 1, the> \HOh,,blr tlll'l(JllO!llt? 0I

1. Nnme fWO 11,,,,t,1\ u,l \I Ill,,, Iii pr Of! lf r.1mcn or thli. p:itll!fl 02
02

- - ------------·

KEY:
01
1. Perforated duodenal ulcer 02
Flvld resuscitation and antibiotics 02
~: Closure of perforation, with omental patch If necessary
• •
Task,;, 21
Carefully examine t he given Item / photograph a 11d answer the
following questions:

1. Nan,cthlsllcm. 01
2. Nan,o ONC cn1~rqenty -.llunllnn whrrt- 11 rnn ht> u,,,,cJ OJ
3 Give TWO p o<il () J")('ll\llvo• lncll t.lllon• r1ir 11, 11 l(lf 02

-
- ----

KEY:

01
1. Three-way Foley catheter
02
2 • For bladder Irrigation In hematurla
3. After prostatectomy or TUR for bladder tumor 0:2
, - ,._. , ~...- ,. VJ
Time AIIOWl!d OS m,nutr-s

22
For Candidate:

A young footballer sustained on Inj ury to his knee. /\n X-Ray wos done showing th~
knee joint.

T ask :

Car e fully exon,l ne t ho given radlogrnph / pho\ogroph and answer the


following quo tlons:

-------- -

KEV:
01
1• Transverse fracture of patella
01
2 . Tension band wiring or patella
03
3.
a) Stiffness of knee joint
b) Atrophy of quadriceps
c) Infection of bone
..
11n1c A IIOWP(l • o:; m1nutr.,;

23
For Candidate:
Matr::>
An lfl 'tCtlr Old m ot orcycllst struck his hcild agi>Jn,;t an electric pole Ht drov1 lo .-i
hosp1tol emergency L<1tc1 on he bccilmr. unton clous The X Hily hoo; br•pn • hown
.ibovc

T ask ;

Carefully C\un~l!'I<' lhr (Jl v1'n 1,1(1100111µh / phutoor,1ph and clnswcr lhc
follo\vlng q1H.''-llOn<.

\ \\lt,.-it I<; hli. ,,, t'h,1t1h 1ll,111no $1 01


"I \Vh.:it ,lie th• \ It I\ I 11\dln I / Ol
3 \\lh..il cltnlt,,1 ,c, ,1r111 !olH)ul(I I Cl n ) OJ
4 \Vhat onol hcr h11p,11\t1nl 11111 lf I llon •h d d oe1 o,
s \'vhat dt•ll1'ltl\•~ l! c.iln\el\l 1101110 I (IOI\ ? Ol
-. -- -


KEY:
1. Extradural hematoma
01
01
2 . Fracture of skull
01
3 , GCS 01
4 c .T scan of head 01
5, eurr~mr
~aleJlQ.d
drainage of hematoma
. . . .
. ·-
T a s k -: 25

Caref ul ly ex am ine th 0 give n It em /


Photograph and
fo llo win g qu es tio ns:

I N ilm c l hlc; llt •m


<; ( l•p s C\I l(l t;( •r lln n In
Im I
2 G iv e l hl•
- - - - - - •
' ll I II Ill • Ill
------------


KEY:
r
1. Two way fo le y ca t hete
2. w ear st er ilized gloves
a) Clean th e glan s w ith
an an tiseptic;

ur 2% xy lo ca ln ge l In to the ur et hr a
b) Po
tip of u re th ra cl os ed fo r S m inutes
c) Hold t he
d) Lubricate tip of ca th
et er
a tll l th e fr ee flo w or ur tn e Is obtained
e) Inse rt ft In to th e ur et
hr
th e bu lb w ith s- 10 m l or dlstllled w at er
f) In fla te

g) Connect It w ith a ur in e bag


MDD::> r-1na1 t"roress1ona1
SurgeiatJ1
Annual Examlnatlons 2009
Objectively Stru c tured Performance Evafuatl o11 (OSPE)

Static Station 2
Marks : OS Tim e Allowed : 0 5 mi nutes

For Candidatei
A 35 yeors old rcmo\u h,, .. r11•• nl l'd with 1 hl•lory of recurr~nL cplsode!i at upper
abdominal p.iln and 11,•r l I I ·, 1h1rln 1 ''"" of th p1!1Xlr .srf! o,; rotlows.
-
Total blllrublt'I -
fOVl'I

•,
1
111'\J/lll
Direct bllirubln • \ "illl!J/lll

Indirect billrubln I tti\1(111

SGOT 4 7\lf/lolt

SGPT 4 IU/hlr

All(aline phosph.:,u,se Q~Ol ll/ htr

Task:

carefully answer the following questions:

01
1 • What Is the probable diagnosis?
Mention two further tnvesttgaUons which can help In reaching a djagnOSIS, 02
2· be ttve 02
3 . G111e ONE method of non operative management whleh may cuta •

KEY~
01
l . Choledochollthlasls
02
2. USG, ER.CP 02
3 . Paplllotomv
27
For Candidate:
I )

A 40 year old man has presented with Jaundice ERCP ht1s been un.,ucr-cssful This
X•Ray shows on altcrnc'.ltlvc lnvcstlgotfon dono

Task:

Carefu lly examine the qlv<'n ri'ldlo(J rilph / photogrcJph and iln'iV/('r the
fol lowlng questtor,<;.

l \\'hat lnv..-~t lQ,Hk)I\ h ,. ht! II 1tono? 01


2 Nan,t on,• po,1l1v,, fln11tn11 1n 11,11 fU\ f I !,
3 \Vh,,t ,,,,rt (If 11111,lly 1'·' 1111 h, 001 I o

-------- -

KEY:
01
1.s
1. PTC art of extrahepatlc blllray passages
2. Dllated lntrahepatlc and P 01
3 . Olstal CBD 1.5
4, MRCP
Ta sk : 28

exa mi ne the giv en tub e / ph oto gr ap h and answer th,•


Carefully
fol low ing ques tions :

01
u• ,
2. N1.11ne tw o the1,1111•u t11,; f11cllc,1lln11·, ror 11!1 01
1111, to r1•u,11nq throuQh 1, 01
3 N.ln1c on e co rnp ll!\ tln n 1h01 r 1n '" r111

•I . N.SIY\(' On t• dlAQ flll lh lll! lh IIII HI


Ii,, 1111 II rJ n1
--------- --------------·

01
1 . Nasogastrfc tube
. 2. 01
mp resSlon of upper GI In lntest lnol obst Ol b
a ) oeco 01
ning etc
b) Gastric lavage In upper GI bleeding, poiso
01
3. Aspiration pneumonia etc. 01
4 . Measurement of gastric juice
For Candidate: 29
~::rsso year old lady has presented with this slowly growing tw,,llh.J iu'lc•• lll"-t ,,

Task:
Carefully exan1lnc the Qlvt•n pnt lrnl/ photoqr<1ph and answer the
follow,ng questions.

1 Enu,nerut-: lour t1llll•11•n11.,1 c11,,qc1r;• 02


t hclpf 11 In , onlirmlng tile <1111gnosls
2 Name on1• lrl\11 ,ll\1,1111'11 ,~IH{h r:tin !Kl ,n
• - 01
01
3. \Vh.ll tS the C(\11'1\ltll\C ,I tun,01 t lhlll Sil ?
on Of u, tumor. produci09 a
4 \'Jh ch 5l1uctu1~• con bil ln1111cr1 c1u11r,o the ,
01
cosm.:-tlc dcll'Ct
------------- ------------- · ---------------~-----------

KEY:
of submandlbular gland, llpoma, lymphadenopathY sebaceous
1 . sanvarv t umor 02
cyst 01
2 , FNAC d 01
3 . Mixed salivary tumour of submandlbu\ar glan 01.
4 , Marginal branch of facial nerve
For Candidate:• 30
A rnotorcycllst was Involved In a roadside crash. His X•R.ay IS shown hrr,.

Task:.

Carefully examine lhc given photogrclph and answer the following

qu~sllons :

1. \,\lh.ll IOIUtY d~,,,, I hli. , 1IIY hnw/ 01

02

XEY:

01
1 . Dlslocatlon of C6
02
2 Paresthesla & qua~rlpresls
02
• Cervical traction & cel"Jlcal collar
3.
For Candidate: 31
A young man sustillncd a pcrlneol Injury one month i.lgo. This 11 h1~ ul,•rl'1r0<Jrai'n
now.

Task:

Carefully exc.'1111lne ll1r (llVl-.n r11t1toort1ph / photo<Jrr1ph und dnswer the


following question.,:

1 . \\/ h.:,t lo; th\' cll,111111'' I -. 01


2 . N.:lmC t,vo 111,,1c ,1 h •,11111 1 with wt11c11 11, 1 flli nt 1 pr icn~
11 01
3. Nam~ ll'VC' cc-n,,,uc ,111,n• th 1t I n ot•'" I , r on 02
4 ~ N:im~ ont• int tho,1 c-1 tr ..,,lin, nl 01

---- -- -- --------------- -------------~--- ---------·--

KEV;.
01
1. urethral stricture of posterior urethra
02
2 . oysurea, retention 02
3, un, Bladder stones Ol
4. Internal uretherotomy
~
For Cand idat e:
32
~I?
A 30 ycnr Old mnn w os t, ll by \he bumper of a motor car Th1. x Pay or his Ptght l{g
I<:; show n he-re.

Task :

Carefully Q)(ilm ln th glv n radlogr;,ph / photogroph and ans-wer the


f ollowing quc .,tlon4G:

1 \\'h,ll ~I\ lh1• llnt11n I \II :( Hl1'1', 01


2 . II l h 1'1-C IS ,l 1,H I lh'll \\it>\llll') I II f ,., .,~,y1 ri m t 'II steps In t h"

t•~atn,~nl 02
3 . NafT\l' n\10 \~"'' COIH()IICat lons lhllt n r 02

------- -------- ---- ------------ --------- ----- -- ----

KEY:
01
t Fracture of tibia 8t fibula 02
• wound tollet and appllc:atlon of external flxator 02
2.
3. I nfection, compartment syndrome
For Candidate : 33
This Is a forty five year old lady who has prcsentf!d with an umblllc.,1 sweJllng

T ask :

Carefully exarnlne the given photogr.:iph and anc;wer the following


questions.

1 . Whilt ts. th~ p1obot>h..• oloQflO 1,-, 05


2 . Whilt signs \'IOUld you lt>O~ lo t n,1 I ow wout J vo,, hc'I lh m? 01
3 . \\!hot con1pllc.:,t1on l ,n ortu1 7 1.s
4. \\!hat ore slqn~ ol '-It nyul \Ion? 02
r $ •

~--------·---··--·------~-------·

-
KEY:
o.s
1. Paraumblllcal hernia 01
2 . Reducibllity, cough Impulse ctl ...-ngulatlon 1.5
tlnal obstru on, ~u'" l
3, lrreduclbllltv, lntes 1rreduclblllty rto cough lmpu se 02
t tr,aease In stze, ,
4 . Continuous pain, recen

rolected with tnat version of Visual Walerrriark, Full ve. sio~·doesn'I ul this m;;tk
For Candidate: q
34
A 17 year old boy whose photograph Is shown here h;)o presented with a neck
swelllng.

Task:

Care fully exam lno t he given pnllC'nt / photograph and answer the
fo llowing qucsllons:

1. Whol Is the nrobnl>h, dlnuno,i.11, 01


2 . Nome THRCC lmpor\"nt IIIVI' ,111, 11 hill whll h f"" w oul<I !lo 02
3. Ir th(.' p,1tlcnl , .. "' n1•1lhyr\llcl, IC lfn ' "'" 1,11, •!ll !Jl'l ,, ' " In t1,111,t1ntffl'ttJI
02
------------- - ---~------ ----------·-

KEY:
01
1. Multi nodular goiter can FNAC 02
2. Thyroid function tests, thyroid s d'o subtotal or near total thyroldectomy
3. Make the patient euthyrold , then 02
• -- ·· - --~ · -~~•••••••u ~• v1 1i, 4.UV U
ObJec tlvcly S t ru c tured Pcrl'35ancc Evalua t ion (OSPE)

Static Station 3
M arks: OS
Tim e Allo w ed : 05 mi nutes

For Candidate:
A 31 ycors old n1flll' f)Otl11n1 11,,, Jlff",l•nll'tl Wllh l(lllfl ,so,,,,,,, uppnr llbdOminal pain
radiating lo lhc b,l{k Ill<; ,1hrhirnln1t ,,,, 1n11n ,1,,,11 r.how 01•n,.r,1h1~d tf'ndt1rnes$ and
absent bO\V('I SC'llll'ld~ "'"' Int, lrtv, tltJ 1111111 ,,. I toll•,wa:

Scrum on,, 1,,-;c? I '(l!ll ll/1111,


Unn;,ry ,1n,"l,1,l' l'U 111,,
Serum c.:il<.,urn <•rH\J 111
Serum bllirub1n ln1u/ llt

Task:

Carefully read the scenario and answer the following questions~

1. \\/hat 1s the probable diagnosis? 01


2. Name TVIO further tnvestJgauons which wlll hf:lp 1n the dlognosls 1.5
3. Name ONE m edicine which can be helpful In tn!at1n9 thl~ cond1tt0n 01
4 . Name ONE compllcatlon which need11urglal tnter.;ention. 1S

KEY:

1. Acute pancreatltls 01
1.5
2. USG and CT scan
3. Somatostatln or Its analog
Ol
4. Development of pancreatic pseudocyst 1.S
For Candidate; 37

Jask: Mat<?

Carefully examine Lhe given suture/ photograph and answer the


fo l lowing questions:


1 . Is thl~ s l n1c h11il 11oyn\111•\11 nr nntu rnll 01
2. Is t his sulur,• ,\br.01hnhh• nr non ,111 ,,1111hlfl I 01
3 . Whot ls tho ll ll\ln t1tl V""' 11)~ " ' 11111 l ll lltrll7 ,.~
4 . W hat IS lhtl d i ollvn111,,u1• (ii l hl!I 1111t 11a:1 I 1 .,

·------- -- -------------------------

KEY:

01
1. Synthetic
01
2. Absorbable
3 . Delayed absorption time
1.s
1,5
4 , Poor knotting quality
38
A n 18 ye a, ol d bo y w ho
pr cs en t~ d w ith n ft11nk po
ln un dr rv ,
X • ra y Is sh ow n he re .

T ask:
Ca re full y cx<1 1n ln c l h c q
lv cn , i1clloqr,1pll / ph ol o~
ro ph und un'iwr•r th t
fo llo wlng q UC<illO n '>

1 \\l hi Jt ln vl \Ill li on h , h
n !Jon 7 01
2 \\ lh it ,1h no r1 n Il l\ do II hQW1
01
3. \\ lh 3l IS lh l I" 11I> 1111 \t11 I I !yin (
I 01
4 . 11 l1;t\ un lt' C llh I \lo h ilt
nr n 01
s \\ lh t ll tm nl houltl t (I n 01

----------

KEY ~
0\
1 JVU 01
2 Right hy drou reter & hV d nt>ph• osls
ro
01
3 u re terl c stone
Ide 01
renal functi on on this s
4 . LOSS Of 01
5 u re tero lithotomY
ol isl1nl \-Jelc-rrnMk•. u Iv ,, I ul lh l~ mmi<.
• e
Fo r Ca nd id a e: 39
Ta sk : M at l?
Ca refully t he 91vcn Inst rum ents / ph ot rt1ph and a
og
nswc r the followlng
quest ion s:

l . ldc ntlf~' l h l. 1 I w O ln•,1111111, ,,11 1 01


2 , \\lh l'I ,lo ~ •lit II ' l,l , h CINI 0 1 l h 111 1
01
3 \\'h y? 01
4, Ou tline c: tr .11n11 uso l 011 l orn 01, In & II ut ():,

KEY:
01
cl.snips
1. Cr ushing & non cruc;htng lnl 1e •,lln JI
d on the loo p of lht • gu t , to bl'! ,cs t•ct ed & non crushing on the
2 Crushing Is use
Ol
on e to be ret ain ed
cla mp occlu de s the blo od su pp ly bu t docs not crush and damage
3 . Non cru shing
Ol
t he tis su es
lay ers bu t mu cos J 02
4 . Sln gle lay er lnc lud lng all t he

• k. Full ver$lon doc5n·r Pill llll~ mtr.l\;


e:
For Candida 40
,og ,.,by
,
WhUo oJoy <ti i l , ,~ -
' " " lh " oOJ,,y
hAe r1e6. YOON; old boy ' " ' "

Jask :
Ph " " " , n . w . , <ho
" " '' ' '" " / •"•WUm
,n , . <h o ,, ,. , ,, " "
ao o ruu y
C/l o ~v,n q q,•
u, i,u1n1 0 n
fo

n urr,<J1
J \V~ t boo ~ In ju ry h i! • J
me n .. o er 111 rnont or,1 I n
2 N " ,
tt i
mpu 1 •11 ,
3 Name n v o co

---- - ------

KEY:
01

ra ctu re Sh a ~ o f humerus 02
1 F
S t, II ' Pla
ting te 0 2
2 I Pla s te r C i>
c.tlon, stress frac tu rc ot pla
m a lu nlon
II lnte
a c ti o n a n d
3 ,.• Distr
41
/

Mat!?

..-z. ....o
V'I N

0 ..
'

::J 0
. . ,0

- - - - -- --
For Candidate: -~------ - ----------- --- --------- -

- Task:

Carefully ex am in e the give n photograph and answer the


following quest ions :
,... ., "1 '
.. .., ,,,_,• I ~-4z .. ----------
,~~-- - . -- - ...
-

• ·1
- ' 1 ft
.I
• I• .,. f,
'
I •
• \! /

Tas k i
c.,"' fuJIV ob~cr-v th~ or-ovlde?d t,h o t uut , tph l!tnd • l'ISWCr the fotlow1n9
q uestions :

1 . What Is tills nee~ swclhn1> a ,11, J • 01

2 . Give the possible co,nmc,n Qluse7 Ol

3-. What a re FOUil p.l'-Slble pressure pr6entatlons oo 5\ICTOUrtOmc} 5tr\lC.lures?

02

4 . GJ11e preoperative pre paration In toxic ~tlenl , 01


M 8DS Fin ol Prof c sslon o l ate
SURGERY
An n u11I 1Camln11Uo n 1t 2000
Objcc t lvc l y S t r , ,c t11rod Pt,r f orman cc Evn lua t lon (OSPE)

Mnrks : 0 5 Tjm Allow~d . OS mln o t ,

For Exan1inoc,
KEY:

1 . Golt~ 01
2. r0c1,nc dend~ncy OS
3. Oyspnea 02
Dysphagu
Hoarseness
Homer's synd--ome
4 . Tab. CarbJrnazc,fe 20-30 mgtdav Ir, to~ dl.Vl(led dOses 0\
Tab. Inderal 160-240 mg/doy 11'1 t hree dlVlded da2S

• I

-
.- - :'.'"t ___ __ -
"" '• A i- •- -• - .- -• .. .-
- ·s: <•
' "
· -- -• • -~
--·-- -----l
·c.~~!I'\- ob sc rv ~ ~h provld Qd obJoct / ph c.,to9r pb nd •nsw . r th
• .:~vii::~ qu estlonS!
\.,ta ...," I
, '1!. IS ...... 0- .~ • I
• · • ... . •

_ G'.v.! :-• • o ma lct'ttc,,s for lb. use?


01
• \'. 1:.-., t:• pe cf , r ~sthc:slA 1, req uired to use It?
C:l
t; Vv.1:::t .,,'"£ c: m~l!clltl ons of Its use?
• •

• •


• •


;
-


• •
• •
B BS F ln a l P ro fe s s io nal C?
M
~
SURGERY
2 0 08
A n n ua l !l lt a m ln1 tJon1 E v a lu a ti o n (O S P E )
O b jcc tl v c..:y S tru c tu ra d P orl o rm 1 1 n c •

sintJ, , sta tion 5
••

TJme >Jlowed: 0 5 mi
l' h •: ~ : CS

. o r ~xam tn0t:

~ (E f.!
- O!

2. \J ." .C .) n ~ .: S .p f nt

G:;.n~~I ~ ~ es th ' !S

01
3 . Gene ral 1 , .;S • le cl.>
4f'8;, ::! \i ,t y Inju ry •n cf tr 6 ch eal lnJurv 01

t , - Fa,!t.::-~ to In tu ba te
us
" - Ir.tu~at:Jon o f 1:sophag

, r ~'°
9-< Clr.

Il._....,j
!) .:l
/
( !a

..
-
• -

- r~i'u!,1 0.):.L • ~ t he pro vid <'d p h o.oa1ro1ph tand 0~1e:r the follov,in
csti;>-s:

-· \' ,iat s thrs lfl• est.,gatron? 0 .5

0.5"

•• \I n..-t ..ire r,r(,n rry bl,rddcr ~tol'IP'1 P o.s


, • v-:try secondary blidder !>~oner. rorm? 1.S •

• • Wt,at ,c; ucotment ot :icco.nddry blDdder ,tone:.? 02

••

• •


f"1 a6 S t-l n a t P ru rc s ::11 u11 n ,
V •
S URC:4JY
A nn ui ll exnm tn nt io ns 20 0 8
t r uc tu re d Pe rfo rm an ce Ev~: ua tlo n (O SPE)
Ob je ct lv c /y S

S ta ti c Sta ti o n 6
: ,~ r l1s : OS Tim e Allow ed : OS m in utes

L=or Examio~ r:•

! . Piat:-1 r,1m

05

0.5

1.5
~ • •r,,, ~ :m=- rt .,. _~ccr src
n ~, c.u rs
ft,,) - :n ti': ~ p~ ~ n~t: ::t ,n, .ct.Io n,
~ - Bladder outfl c1•, o tr1Jct,on,

(t!> - Imp..1r ed blc.1 oac::r empty1n9 or


t/) _ A'To;-e,g,; oody

d. 02
s. Underlying a.use so..ight and treate
'
~ ) • Ut ho lap ax y , 1y . •
(lo - Percutaneous suprapublc lltho11p1>
• L
~ ) • ESWL, :

• I

'

• -- '


;

..... .
) I.
\
~
I

--·
....
- -----
: .. ":! f.Jl ly 0b sc n1 0 th o pr ov ide d obj ac; / ph ot og r~ ph •n d 1: ns we r th e !
::, ~t v1ng q1.1c.5t1c;. r1s:

0!
:. ,Vi l:: Is ttl ,4 cbject">
O5 , 0.5
=· Nn 2re It I!; 'u"""f.l ., tr, which situation It fs us ea l
rer consideration ls CheCked? O.S
distal pa ten cy or the ~tr uc \ur e un<
3 . Hov.• 't

re mo v~ . It tnere n~ <llSl31 obStnict1or.1


4. On Nhlch Po:..t•operat1ve day It 15 •
01

a ne wi ' h dis tal ob i;tr uc tlo n It. ·datactL'<I post•oper~tlvcly, how tha t
,. ff sto
l.S
can be rem ov ed ? •

• ,
• • •
• • •

r -1
1

"II
MBBS Final Professional iAate
:. ~-.1
,,.·i ../
SURGERY
Ann~III !xamtnatlont 10()8 0
Dbj a r;tfvu:y St r ucturecl Porforn111nco Fv1tluatlon (OS. E)
·""•·yI

':/ Static St ntiQtl •5
Mo r t:., as • T.hllo Allowed: OS minutes

\ 01

2 . A. ~r common tno c:uct CJCplonJ.t~, ~v y trJ artt n:;~


I C ,-$t:'UC'tio, Ol

! O.i

l 01

1.s

• •

' .

.., . • •• .-

' • • {

' ..l
••

..

I
1,

l
-
- - - -. ~ ~ .h - ,S j:l""t.~ ~ - ... • •----• • ......_.,_. • ,..,. • • • • .-,.,... .,. .,.
~

I • t

. . ,_ , . - r

--

-- - . - -- - - ---- -- -
For Exart't iO ..,'u.
o uest fon .;
0 !,
1 . \\' l- t 1n, c,,t,g11 t10n?
01
2. \\ h. lt .ll"C.' lh c I n( l ng \?

th ,. , 01
3 . \V na t ty pe ot lr. ic t un : I:<.
01
tr u; tu rt' ?
4 . \Y ha t is tre at m en t c I tn1
utJons?
1. 5
s . Name any Ti i RE E c.u rnr,h
K E Y:
OS
X• ra y 1 rm AP an d la te ra l
vie-,..~
1.
e pa r.•.ln g th ro ul )h su pr ac ondyt;,r ar c. l or hu m er us
2 . Transverse fratlUf<! fin 1.Hspl,l(('d W llh ov cr l,l p 01 tro gm nt s. O1
Fr ac tu re s c1ro C'OfflJ)IC'tl'IV
u, . 01
ct ur f' or the - hu m l•r
3 . Supra co ndyo lr lra
01
tr.ium,1 by 1\1\CO f
4 . Ear1y resusclt111l on of n ol re du ct io n (In t er na l fhtil \lo
n)
Redu ctJ on an d co nl lrm ol lo
ln ed by two cr os s w h,e$
Redu ctio n ca n be m oln to
1. 5
ry ) In ju ry
s . Vascular (b ra ct,lal ar te
N eu rologica l (radlol) Inju ry
ct u rc
Volk m an 's lsche m lc co nt ra
Mal unlor1
t

: 52
I
I
:1.ll• Ol •OO )
"' I
uo
._s_t_"_ _ "- I 1.
_n_G I M at t?
- )

:oro, alonnl
~RY •
,u on, 2001
:,,,.,,,o tv11lu111lon (O S' ~) . =
I
~11ooct. (1_~
,I Tr,.,. A l ~ s 15 ml r,ut.es
I
\
I • l
I
1•
I
-
I
I


I
• .,
I l
I
:CT ecan / pb otl>!.;ra ph an d

'
I

I
• i•
I
t
I
I
I
I

01
01 •
,. i
I
~ ...... 01
01
I •
t 01

' •


• •

, •

,/ ~

,/

, ./ •
Mc:>tl?
• •
MBBS Pl na l Pr ofd tsl on al
SU RG ER Y •
An nual l!,com lno t l~n • 20 0S
Ob}ec :tlv o/y St ru ctured Po rlo :-m 11n ce •
6v alu• t on ( OS PE )
I
• •
t
lar k :Os

i:o:- ;;x,amlo§ci •
- II
:;<t..,::Y .•
...

i
I

1. er $_Qt\ 0ta1n
01
2.
3.
Fto ."\t oP l(te ~I t >ctr~urel l'IIM'ICOffla
Mi~Ole ~~~ 11Ury
oi -
01
4.

01
s. ABCOE cf trau.-na
01
Su rgk :ai em~ Jt "',""{-cranlcllDmy lfld .V I CJ •
LltlM fl ltl I NID=III
ICU ffl3 .~~• M ll' lt ~ - 1l'lltlwly


• '.
• •
..
4 •

• •


,
. '•
54
r ...--~ -----
"' .. .,____
'?c
... 0
0 t::
~J!
.. Ill

.-
I
--
Ct•,


Taski ...
'
answe r th• followlu g quutron: . -
Qrefully obMr ve tlw g iven set of l offlUIIMffU / p h o ~b a i,d

0-1


• •

,., \Vhilt is this aa111 ol tft.CheoallOmy? t .S

._,.• ;
ti
l~.-,I
MBOS Final Professional
ate?
SURGERY
An"u-'I 11Ju,n'llnt1tlon • 2 008
Objoc t i vol)' Struc tured Performonco Evotuatlon (OS PE)

static Station 1
Marks: OS Ylmti Allow ed: 0 5 m lnu

For Examtnoc:.

01

2.
a. ~ l!I' airway obs:tn, lei
b. To ~ 08d IO{:~
IJ(le
C. -'S p,1dlmlM,Y \0 C~ l ~l),n ~racJons

3. AnyTWO 01
a. ~ IP!{l'IC!'Ttlage
t,. Pnei,mothorax
c. TracheAI stenws

4. 01
a. HumtdlflcaUOn
b. Tracheo--bronchlll tollot
56

- -- -. ---- -----· -... -.g.iv..e.n.. ..p h(J\l)t" ' ' p h .no nsv,Ct" U i• follow lns
: c lc !:y :> b $ C l" V th o

~.-• ~
") , a .- -- .
.;;+;.~t ...~ --s·
01

01

c1n :i 9 ernent or thl5 p11t1ent?


3 . 'iti e t :s lr..t, al m •

c.om pllc:at1on1 or th is cot1d 1t1on? I


4 . f'.;,me TV/0 fate •
I
••

•\ •
• t ~

• • ' • •
l.
'
l
• ••

. • .
/ .

, I
/ •
,I
~
11atC?
,.% • Maas Fln•f Profu1tonal •
SURGa!RY , ..
• Annu, 1enmln • tlont
:ZOOS
., ObJoetTvol y $ t r uc t urad Parlorm, nu 6v• lu•tldn ( Ofil' I!)

st1t1, station .2 •
J•!e:-ks: Q5
I
-i::,r sxamtnec;

01 !
..
.
~~ cb:c~tudor;:.G'rl~lipplfbl:k-.Lci-.blwc. . . · 01 ,

• •
• ..

.. •

•I •
• J
For
Task:
carefully o b.serve lhc given X· r ,l '/ / ~ltn t ogr•ph ,n"td , I\SWf'-f th~

following qu~st,ons :

1. What Is u,e &bnor m..,1iry presenl?


Ol

2 . What a re trH' cornpllcattons or l hls 1tocturll1 Ol


59

~i:)
MllBS Fln ol Profess ional
Mat1?

SUflGERY
AnJ1ua l l!wnmlnntlon• 2008
Objc:c t l v c ly S t ruc:t11md Por-forrr,nnc o l!vatuo tlon (OSPE!)

tiJllQD 0
M arks : OS
rlttto At1owe1d 1 05 m i nutes

For Examineci
KEY:
02

2.
03
• vascu111r ln)urv
• Neurol09lcal Injury
• Maltmion

'
u
• A n nua l Exa m lrta tl on s .t uu a t io n (OSPE)
e d Pii @ rm a n ce E va lu
O t,Jc ct lvc l y S tr u ct u r

- Sts1tic statir-- a
Ti m e A llo w ed c OS m ln u t

th • 1n1t, u m e ft t / p~ r- ..h pr-ovl d ed and a n1


e nrc :utly o a m 1 n 11

t 1 .i ;~uc,..-lng qu tlon,r •
I

,:n , OS
1. . .: ~t ~ CM lMtn.:
••
rt '...
.o.s,o.s
2 . a1rat e de>wn Its di ffe rt nt

' 1 ,S(O.S earn)
ri te do v. ri THR!; E ln dl cl ti on l ot ltS us e•
3. W

• •
~.s

l1 In st ru ment ?
4 . Ho ~ ~ II you st,e ..
rillz e tn •


Mat!? •


~ • MBOS Final Professional J

SURGERY
Annual Examination• 2098
ObJactlvoJy Structured P•rfor,r,1nc:• evaluation (OSPl)

fitat1"scat1gn 3 •
Tim• All,OWt ds 05 ·mfr,utcs
t-1::ri-s: OS -
'
T· ·,S\si

c .1e.stions:
01


02 •

Ol

01
,
,

.,. •
'- -- -~ ---~ .... .,•-· - . - -..n--·-
-.. -. .. . -
- - ~ - - - --

I .,,!c 0
C\ II) •
~ •

o: ;J •

I ,:. ft
.. ~'l

- ·- . ___.. •tt r: :•- ,. -· •.::::::t\

_ ... ,.,
:-s .
4'- ..

n CT Sc an / Phot og ra ph an d an~w er the


.:Z:-- ~ H~· e., a m;r: . ti"ie give
"'o!!cw l: :n ::;ue stJc n s:
01

is Image?
~. W:iia:t :s th e ab nt ,mollty ln th

en t ln th i. patient? 01
fca l feat .ires m ay b6 pres L •
,,, V/ '"tat c.ii r1 •
Ol
w :ia.t Is tti e tre ~t ~ent of ttl lt abnormalltyl
;. t • •

• •


..,...., ·._.--..... -n•
,;nn ua l Exa1111Qat to 2 0 08 lu a t/ 0.1 (OS.,..,E;
c ty St r u c tu re d PeYfd rm ance Eva
• o~_·::: ~ tlv

s t a t i c St atton s ta tG '

J\, J : 05
- T im e A fl o·N e d : 05 mln u

f 0 '" ..:...x·· ro tnec 1..


J
01
.• -, s ,s e' r x .n ~ f '\lil,
01

,
• 01
-
; ;;.

~ o.n , ~ encl ,....... - - . . . . . . . . ...



.,. t't.

• •


- - - -\::::fl) . 0 \

I
~
I
'<"


f, . 64
• 11 •

'
I = a
' F h = M

••
:,
-
~ '
••

·-
· 2sk!
-
{ a:2fu!ty e xamir:e t he g iven object / phctog r?ph and nnswcr ,
~ !!owlr.g qu e~ ·lons:

l. Jde;nttfy tnrs obj1:ct? 01

2. Name its p~rts? i.s


• '
3 , Write 11-tREE riidlcatlons ot this lnstrumtnt's use? '
l ,S

4. How Is this tr.strumef'lt $terlfl:ted? Ql r






I
M@fe
MBBS Flnal Pt otesslonal

SURGERY
'
.

An ni.I.ii Examination• 2008
Objecti vol!f Struc:ured Perforn:•nc• Evolu,.tlon
'
(OSPE)
.,

• Static StntlQQ 9 \

M i.-l<"s: OS - T,me "'lowedi 05 mi nutes



t

- ..
;E'" •
· --
OS


•• a ) • IJ-.,per ai~ ~ ~
t) - Un« --sc10.: l patient
C) - '/ £ 1 \ ~ svpport -
I

• • •
....... -
---

For ca n di d; tte ;,

Ta sk :

ca re fu lly e xa m in e th e gi ve n sp
ecim en / ph ot og ra ph an d an sw er
th e
qu es tio ns as ke d by th e ex am in er
.
"""\Ml'
Obj •f-lW,Jy !:tnt.m, l'IHf ~
rMfl~Mlt.~• ~
" ' • ~ 1"'8tti ~ '41 ..

St1ttQn 111 Mate


lD~rJl~d

l i. Nnl"tl '\\"° h\<. 1 ~\f\tf I l to. lf\fi "'"""' I


l
3 , \\fMt ;\I U\ l\\f\ N ~• \ • I

• ~ Wh.at -~ rttl\ •
1u~
1, ~!\Ohl ¥Nth te4'<,\\lt~
c,
2,
•.Q- Ofl tMt.-tn
b..1,.-v l!ffb\
\.

1tctumllfl
_, ,, , __ _-·· -······--·
]c ct l vo f:, St ru c tu re d Pc rl<t5'aDnc e Ev alu ation (OSPE)
C!J

· S ta t ic St at io n 5
Tim e All ow ed : 0 3 m l r .Jt e~
M ilrk .. : 0 5

-
Fo r l§xa m in~ r~
.
.......
KE ' _:
.___
05
.. Pro1.t<>SC'Op..

01

l ,S
3.
.
c: ;·· tl h e ~ : pot~ a,l ¢r - cant •
~ -~goo s to .U!
.~ 'f.
~ ] ',:-' !P~ t ; •· ln!cctlon S d e ~ PYtor ~
01

a. Ac:J!'e nssure ln ..!l<'

':. Pe.iana: ha.rn'"tc.ma
01
s. Ji•• wdavfng
• \
• • • • i
r•


\

• •


• •
..
..

\ - •
I
• -- •
.'
~

'

... ____ ___ _ ·~


~ S K:

••• - • - •••••••••••• • ~ ~ - -
·--- ---------- ..
::_ref .!!ty cxilmlrtc tho provided o bj cict / photogt•:,tt ond Ans-wer t t
-...::!olvin J q :Je.sliions~
1. Na,,.e the object. o.s
2.,-Na~ the ~rtS. l S
?,, GIV~ THRff OO!T,m on ln(IICdt1ons of Its \1$<: , 1,S
4. Giv:: n~o o;,mmon complleatlons. o.s •
$. Give Tv,o mcttlods or relfltvat It Its balloon doeS not deflate • o.s. 0.5
• •
• • 1

• ;·
•••',




• . '
••
r

,;,
• •
' •
.. 1
• "•
'

.• • •

• .. •

'L
• • • i
" ~
MBBS Final f1@ fesatonal
SURGERY
Annunl E!xnn,l not1ons 2008 •
Objectively Str:.icturcd Porformonco Evo/u:,t}on (OS,fTE)
.,.


Static $,t .a tion 1. . •
M.: ·ks :OS Tim e Allowed : OS ml., utei

.E.c ·i E'<amioer; • -
-K ~Y :
1 . rhrce "'3'1 foJO)~ ~\Ml r. o.s
2. Ttp with ~:.tra, a.UOf.11', Chllt\n«~ rot fta t, ~ Ulf; ~ J« IMQ.tW>n
Sahne/ drugs th:tl'll'lcf tGt ~nkting lO urtn 1r, :g 1,S·

3 . \cute reteJ'ltiOI'\/ TVRPI ~ l>Jlk "It ~ffl'ISJ vrlM ~ . 1. 5 ~
-..,J , {4 Ii/fl--, ,_-. N - ~ ~•
4 . ~:lfecbon, H~•90- .,~.,, ~ ·~ ' ' O.
s. .'.:!lpra-publc rupture of ballOOn with L.P n ~ under'~~ gvidaoc:e.
,..... ptt!re w,th st1llc, Of U· et.enc c.atneter. 0.5 , O.S
• ~ i-;JN,Z ..,, , .- t ~ JI#~'"' ~ Jal c • •
r.•


' ~

• .,,..
• •
r
•'t.

l



~
. '••

I
• • ' ~

• • • ••

• • .. ' •

'

. ..
l •I
• • • :,
' .,. • • "

- . ·~· ·-~ ~- .. -:' .... , ,, ,v,.,, .. ,, ,., ,,,,,,
71
, ,u,1 ;

o..,
2 . Name any FOUR comphcottons ( M<ltcrnal r» fcl11ti encountaed ISrOund 1rw: ttmc ol
delivery 02
3. Mention .1ny THREE problems bolby can fdte tn c.irtv neGN,tAI period) 1 S
-=4 . What other fetal complrcauons c_a n occur In a pregnJf\CY compUc.sted by dlc"lbetes
(Any TWO) 01

----- ------------- --------- -----------

\
,

- -···~--,------·
l Cl~ ~:

_.,._.,..,., .. ........ .. .
--~ -·-------- -
-

~ r _:uJ!y ex.ln~,n~ thu Q IVf.tri obJI:\., / p h Gt ogf itOh .. nd a-n~wc-r the


·o ti ~\'c-in9 <,:u e!.~!o ,s:
.: • 1 ~m~ the object. OS
01
~ ::iv t.: •ts parcs.
3 . C tve .WO 1ndicJf-. ~:., fr.,- IC$ ur,e, 0.'5, o.s
4 . Give 7HREE ~--"· f t.'Qill~1te~ for llvt.ir b101>~y. 1.5

J . ~ Ive -rwo cornr.,one::t compllcat10os.


o.s, o.s

•• •
~

• • •
• •


l,· •
• ••
• •
• •
'•

, 1
} •
• • • f

,• ••
• j
MBBS Flr~ esslonal
. SURGE~Y
Annual Examination• 2008
Objectlv oly Structured Performa nce Evaluati on (OSPE) ••
.,
5tatic station_, •

Mn :-ks: O:.i - '


Tlmo Allowoa1OS minutes

. •or ~x,an1tneri •
-

•• iruc-"'t blOP"V c, 1:(11(1/ c,,r cutt•no b!OPJY Madi~. 0 .5
:. Obtt.r.ll\)f tlaf'dk,, OtHUC;tl<>r, a,tnvl,, hcrid a n At, ·~ ~ for- ~ . 01-
•. Liv¢: biopsy/ O.Oµ.~y or ~ lid u,mowtli o.s. o.s
: • Cor,-eetiO."l of lb.10f'1i\61 QH~ul:ltlofl bV v,U \ r.,
Arr~:')9~~t 'of I:;~
Anli.>1otk:S to l)rovent scpt1;ae.ml4, J'\i~ ~ ~:l<I C¥'! .et u-ni:r. 1.s

Ol

..

..•
• •

• • • •t •
• I

, J

• I
' '
'.

•• •



• ~




•I

\
r

...
_______
.- ,.. ___ _
'
i' ;-.:,•• .
..... ._. "'

C ref:.i'ly cx._,,ni, c t h<. given obj~·c.t / ~h<>t o<Jr ph , nd .. M wcr the



tc ,.:>'I,.:tg ~ uustlon!>:
Ol
1. ldt?anl-,,, the object.

2. Give ·r:vo common operatlol'\r. wnere it Is fn,;ll(atc:ll


0 s. o.s
3. Give TVIO pnnciples or ln!.cruon ot .i drain, o.s, o.s
0 s. 0.5
r. The s~stem pr<:vents whlth compllcatlOn~'> (Name TWO)
01 •
.,,, \ol/hlch sys~m or dr.-itnage 1t ls called'



' •
'
't,
• \'

.. ' .,...' . '
•• •


',,,.\ .
• l r.1aes Fin.al Profcsslonal
SURGERY
Anl\~~I lhcamlnotloha 7008 ... '
Ot;Jocrivoly Structured Porforn111nt:tt EVD(UDtlon (OSPE)
,..
,
5tatlc Statton ,4 V f
I



Mol"k ! OS ..,.
• Tim• Allowcch os· mlnut:!!5
' •
I

l• \

' I
••
r .~V; ,,
1•

t Suct:tt • botctc \\'Ith m111tl1 It !)Cl fqr-111~ Ur l't


. _. Thyro.. .«.t.w:l'fl mo~t ttwny.(l\~y 'TWO) 0.5, O,!S
~ Broug : , n:fu9h sep.11 , , 1111) wound•prc:c~ 11 ... ~~ It! tiotU i ~ '* · •
o,s, o.s
-tt Haem.:. ;.;;n:,, infection, ett,• 0 .5, o.s

,.;. c---::;:i suct:cn dra•n~ e~ ~ ru. Ol •


I •


I • •
f

• i-

,, . • •f•
•'
• {•
• '


_tl a I!
'
.,I
M BDS Flnol Pro fcsslon a l Matt?
SURGERY '
Ann unl C1enm lnoll o n$ i ooo
Object ive l y St ru c t ured Porforman l!e Eva luat io n (OSPE)

Stai:1, s_tc!tioo s
M arks : OS

For ~xamiogo
KEV ;

1.. l M. A. ( UJJ'yflgt.' ~I ~\\ lo. A,,..,.,y I (jf


2. General ~t~ Ol
3. Yes 01
4 . LMA 01
5 . Yes 01

I
MBBS Fin al Pr of cs slo nr .l

SU RG ER Y

Annl:~I Ex ;,m lno tlo ns 20 08
vc l y St ru ctu r ::rl Pc rfo rm c1 n cc t;v o/u otl on (O SPE)
"Jbj c ctl
-
t-1a rks : 0 5 Tlma,Allowcd: OS mln~ (C!:
-


J
T,-.s k;

ca re ful ly cx am ln th 11pcc1n,,.n / 'p ho t0 9,, ph of ttk ll' 'dmen'


nt wi th t hy to fd dfHM e an d, ,.. -. , th e 1o tlo v1:-
r-e_.- no vc d ft"Om pa tie
q ;:~ stion s:


1.5 (0 .5 e. J\)

sur g.c.i' tr~ atm en t fot tt,y rol d tn. ilg nao c:y In h.~"'I fiStc gro-.1p .
2. Su gg< :!;t the
01

What a,e THREE possible operalivt comp


llat1ons of ~Yf'Otd.$Urgety7
3.
' ' 15

the tre atm en t option tor solitary <JIStont bony me ta~ slS ot th '(nld
4. .w ha t Is
!cf-
rna llgnancy?

I
;


.•

, ....

~

- _,. __ _
~
M B B S F in a l P ro te9 e n a l
S U RG E RY
ns 2008
A n n u il l E x a m in a t i o v;, tu o t to n ( O S P t:J •
~ rm ::Jr.ce e
tu re d P o rl o
Oj J e c ti v e /y S tr u c I

• - S t a tic ~ '~ t i o o 7
S m in u te:.
··..cs .· o•-' T im e A ll o w e d : O
1'
• •

j
F, ~ '2 }: l m i n e r ;,
., .: v. •
r,
~ ~
1..., •
..
••

-:.. Th" 'o .d - ~nom


e
l> ;a p .,a,y CA
~- fo ll :. i• ~ r CA
:. . A.1al)l,1S-"e

01
To a ,• u ,~ y . ! s (0 ~ e ,c n )

(A n y T n R.Ef)
a f-'a.e'l'lOrrhage
b . Tension 1-t, cn1a
toma
en t la ry n g e a l n e rve damage •
c. ::lcc'Jrr
d. pa: wthyrold Jns
uftici...ncy

'..
y
c:. ThyrOld Insufficienc •
. 01 •
etc.
• R a ~ lo th e ra p y (E~temal b u m ) • • •
4


• • •

, •

.. .




,,.,,. ' 2 18 1

; .. t. - ~

, I
,, l

/
I
••
i
'
l
- -- '

..

• • •= •
a ~~
• •
t ~u l!y cxamlnc the provided t1dl(ltoo~ t atudy/ photogta$)h 1tnd
,ns~ver the following qucstlon1:
..
1 •
What is thlS lovestiJ;lt!on ciilled? . 01
• '
~

Ql
Wrtte do~vn TWO firidlogs.

0 •
What 1-s your cllnlcal cfl11gnosli tn thl~ coso? '•
'
01
4. Give TWQ causes·or these Mhdln;s,

.
' ''
r •


'
• '
>



1·u-~ ' •
.. Prot~r.slonal
......

IVIBDS Fln.il
'
~ Mate
SURGERY
Annual IJ(amlnatlor,s 2.on11 •
ObJcc:tlvc/y Dtructurod Porform•nco I ,,.tJon ( OSPE)
,; • •
I

t,J"t Mll "t,; : OS Slgtlc Stgtlg·
f '
..wod: OS ,nlnut..:s
••
i •
' f

.•
K
- .::-y
., •. • •

2 . t.t11tipt.-: &!f-thiid l~IS Ind O l ~ 00,)0 ,J lntititlr.c. C.


'
.. C,l

,
::-. P~raly k. 11<!u:i •
b. !wl, ct,e,1fc;b1 ~
etc. •
t!Jg1 • • •
,
st

-
... . .;IQ
r
.) •
.. - \
/
1:l



" . A

- • • •• I
I \
••

'·•

.\
~ .. '
..
..
...,
t. ,.
('
,, .A, -- ,,
....

1
I
•'
. ~~
J

:I ~ • ... )'~-~
- •
i
\ ..• •

• •

- •

' •
..
\


.•16 DS Fina l Protczo10na1


SURGE82
• .... ' An11u111 !Exnml nat lo ns 2008 •
C - :11.:ctlvo ty :;truc t:11r c d Porformanco Eva/va t/on (OSPE)

·- station i !
!oterac~iv.e •

T1mc Allowed: 05' minutes


-
~
'
- ,• ,~~. ~mint~;,
';
.j..C"f • • •
1 "'n\· 1we, (.10,,,11,,11111• 1• ""' ,, .,,., , .....,,..... ")!" ,,,,1 ,.1 111(0 ~ ,-u.:,)
2. y 1.1\ , 11t• 11ll1li'\ l\l 1"' 11 ,u,1, " ' Vt 11111ll)Hil I l•'"ll ;.- J.:,
s. \ ,,..l , ''"' •.tt '" h , -. l~, n111h;nill iJt t Ir V i.~•'' ii ./!Ill''
( , 1entic~ Tf'l~f: only) 1 ~ (O.!) e:~)
u,.;•.., ~4 tit~ 17' • f1Pel'IJ '11?
(11 •


01 {0.5. 0$)
t . C:ld iron, Uln:t; R..,t1'11trford MorrlloCW\ 41\ C.. MY TWO)
l .~ (0 ,5 eam.)
2 . f ...!~ . PeM<. Pre/post 11fltt, w~-ms,.~ic.
1.S (0,5 ~ )
3 . (. \llyT.-i.REE)
z. SuJ:,cutaneous tJs:9.111;.

• •
• ••
t. Extern.al o!)lk1'-' aponeuroJII, • •
c. lnterNll onllque. •
{ . Tl'ifflsversus abdo~ll'IIS musdes. •
••

.. . ~u.tonl\.lm, l ., •
' • O.l •
4, ; .tisorbable. •

,


-- . . •. ....., •• ,.,. , , .__,.,_.. r I

Stati~ St~ n 1
M o r k s: 05
Tlrn c Allow Q! OSn11nvt<'o:.

For Candidate:

T as k ;

Carefu lly C>til1nl11l, I h n nlv,111 )( r 11y / phot,,ornph ,,od .,n,wer t h e


fo llo\.'oll ng ('IUC1otlo111< 1

l . \Vhat '" thl,. ,n~c I QIIIII n • 01


2. \\ ni.,t ,V\' the hod1n9 01
3 . \'lh.11 is <ll tiCI ·111 n lhlJ Xt y r ,m J

4 . \', h.:,t IS early trc.,11,1 Ill of ll'I!, tr;i ..,, f 4J<);QJ 01

5 . \'that Is del n t,,c lri:: tmcnt or 1n1 er .:cs r ctUf' , 01

a, ..

o.,. -
I ~a
0' "
0
...
s upplcmenta,:;. Effw lnatlons lOO lJ OSPE)
Objoctivcly S tructured PerlfilHnancc Evaluation (

Static S@tion 1
M arks: OS Time Allowed: 05 mlnute1

For Examineri
KEY:
Ol
01

3 . Kn~ J01n1 IS 1\<'ll \ l'loll\l , l t VI tli not i ~m'I 01

01

01
O bJCCt lvcl y S t f'IJC(ll f'Cd P crlorm,l n CC l:Vil/U,lflO n (0:SP()

Static stJ\?on 2
Marks: OS T i me Allow d!- O!fm ln u t•·,.

For candidate;
Task;
Carefully observe th r,rovltl~d 11hntour,1i,h i,nd n1wer th a followlng
questions:
1 . \Vh.ll I~ lh~ t.•, l' !.1\111, i!llC',11 01

2 . \\lhJt lhl!. .;l1,1n ton1f1 • l 01


3 . Wholt ,11c C{lfTHl\Oll l~J'CJ, 111 ,,11 Ii luf 'I 01

- ___________ __ ----- -
4 . \Vh.ll :,r1• ~ l\'f t\'!11111'()1\ Ote!;C!Ol tlons f t:.11:c ,.-, ...c111 1 OJ
. -: ,
-- -

I
"-"'UJ'-''-'"' ., ~ ' Y --'"' "'"' ' ""' 11:.! U r ir.: rr v , , ,,.,. • • __ - • •.r~ ---- - •

static i¥S1tion 2
M arks: OS Tln1e Allowed : 05 m l nul

For Examiner;
KEY:

1 . Raccoon C\'l~S 01

3. S1n,p1~ lin ,,, 01


Depl'\"SS ~
~~ of ~\...ult
Orb,l.11 biO\ out lf illChll •

4. Presentations of bate of altull fr.,ctli~ . 02


CSF fistula ,JM-.-z.,-.,,._,... , ,IJ,.e«• 11
4,.f"'·
SubconJunctlval '11~m11t~mc1s, Anosm,a, CSF ~
CSF otorrt\oe.i, Vi l , Vlrt cranial Ot'!rve pals e-s
~ ; ,.,..,l:AL Wl\,i ti1
Objectively St'ructuredf!'lrformance Evaluation (OS P

static St1tioq J ,. .
Marks: OS Time Allowed : 0 5

For E,xaminec;
KEY;

1 . A 1r,'II.>\ 0 1 ph.,r n1Jomt 01

2 . Unron ,('u p,1lt~nt 01

02

4 . Pt-e'II ~l ll)ngU<'! f II n lh Ot
• vu11,;.,.u v c.:1 y J t , u t.. 1. u, c u r cr 1ur rr·1t.1r,cc t=Vilf u , ,tton ( 0 5 Pt;)

Static St~~on 4
M arks ; OS Tlm o Allnw<•d 05 mi nt1te,

For Candidat e;

T ask i
Care f u lly cx,, n, lna t h ( {) hll'III X•r,ty / 1111,,tn r, , 1, p h <'I nd !'ln , w c r th e
f o llowing quostlOlll :

'
02
3 . \\/!'lat arc COfltr;i nc e.:illOI\ ror th I t•1dyi' Cit
05

s . v hat 1n1onruit,on It g, vt•s? Ot


------ ----- --------- ------------ ---------

\
OI •
Oc
.;. 0
0;:;
' ,.
Ill _.
N U)
ObJect·/vcly Structured Performance tvu1uuuun Iv~ ,-,,;,
89
static station 4
Marks: OS Time A)foWed: OS m fn utu

For ExCJmineci
KEY:
OS

uo to bladdrt. 02

o.s

s. Anatomical and rur,ct1onol dct.tlls ot both kJctneys Ot


stc1tic st~R>n ~
Marks: OS Time AIIOV1Cc;! t 01> ITi l nulcs

For candidate;

Task :
Car efully cx1u,1 l11t.1 th" fJl11~11 )( r,,v / 1,hot ogr11ph nnd nnswer the
followin g Que t lons1
1 . \Vli.,1 i'tl' \ II\ hn1t1111J 011 th, 111 ,n him 01

2. G,,l' l t~RlE lll\l ct\~ l 5

3 . \Vli.ll .-ire. l''t)t lc<i I nl(ot hnCl•ng,? 1S


01
------ --- -------·-----·---- ------------

°'"'
<;',:
.. 0
0 :;
• •
l!J Oi

I
Objactl vcl)I St~~i;;~;~·;;,~;,~-;.m ·~~e iva/un t /011 (OSPE)
static station :;
M arks1 05 Tlmo Allow ed : 0 5 minutes

For Examiner:
KEY:
1. Free gas und~r till' <11a1,111,,urn hll ''"' illy Ot

2. Perf~rated duodol\t1I ulcr, 1. S

Perforatfoo of ('()Ion dul.' 10 lllilll!Jfl1u11 nt o ,, 01 f\ittion


Peril!tt t)tll\g inJUl"'I 1\1 lnh l•nAI t111l t
Post l.i1X1rotomv
3 . PertoriltlOI\ 1,s

a) Distended abdomen
b) Tenderness ,nd rebound tencle1r. ~
c) Absent bowei soun~

4 . Emergency 01


Static S QRion 6
Mark s · OS Time Allowt!d tJl !i> m 1nu t 1•-.

For Candidate;
T ask:

Carefully xnn,l n tho glv<1n R11tlloq,,.ph / phntogr ph un~ answer


th e following quc•ir;llon,1

01

OJ

OI Ill

·-•
O c
0
• 0
..
II
II\ . .
l'i Ill

t
static s\iti2n 6 ..
Marks ; 05 Tlm e Afl owed1 05 ,ninut ~

For Examiner:
KEY:

01
2. S1nctu1 •, 01

SIi~ f()( C)t()log ~• n(S ,n100, I ••tnU\100,,


&-u<h~~ trom !>t1 :\Ur fOf 1.:ytol09 ...,onion
3 . Remov.tl oi \t001..-s, $l«!nllng N trictu~• 01
4. Fa,Iure, 02
Ba-cterenua
CholangatJs
Acute pancrellt1t 1s


~· . """ •.,..,. ...'- ... Vt..UUUUUn ( v:,pj_ '

Static ststiftn z
M.lrks : OS

_F or Candidatei

Task:

Carefully ob st-rv,~ t h plvcn 11hotoo,11,11, ,,no •n•wl"r th lollowlng


questions:

01

o,

01

5 . \'ll'l.11t are DOSS Die ,o,nphcllllOrlS C.' '"11, COn(f Uon) 01


------- ---

OI ,-.
'i' C
..,. 0
0~
.s
~ 411

l
Mt,D:> r111a1 .---•v•---· - -
SURg§RY
supplementary Examinations 2008
Objectively Structured Performance Evaluation (OSPE}

static Statton z
Marks : OS Time Allowed: OS minute

For Examiner;
KEY;
1 . (\ophl h,lh1,os 01

01

Oi

5. Exposure ker4t,tis Cornr.al ul~rotton, bllnoncss 01


O bject ively Struct u r ed Perform an ce Eval ua tion (05Pl: J
96
static station B
Time AIIOlfl Cd : OS m inutes
M 0 r k s: OS

For Ca ndida tg:

Tas k:
Car efully obsorv th~ glvt1n pl'\ntoo ,aptt and . n,wer the f ollo wing
quest i on s:
1 . \\lh.it IS. ll\l~ Q!1), 117 01
01
2. Name It 1', 0 Clit(l ,. ,
1)1
3 . \Yh.>l IS l~ ln'\\. 1\ f O(U\ ' lnll "'"' uonf
OS

______ _.., ______ _


-------------- -- --- -------------

[ •
MBBS Final Prore5s1v11°•
SUR~ Y
Supplon,o ntary E,camlnatlons 2008
Objectively S truc tured Pcrlorman co Evaluif tlon ( OS P E)

static .s tation 9
M a rks : OS Time Allowed: 05 mlnute5

For Examiner;
KEY:,
05

01
o.s
4 . BJ~agc- of subc\itoneous lymollallC c.l\th !ti ~, W1"0f o.s
5 . Skin ulcer-atJoo, H-,rd nodul , ,u 00'0t1Mt1 1S

6 . T4 01
·- w , ' ~
MBBS Final PrQJilsslon al
s SURGE"AY
Object i v e ly ;Ji,-plc m c ntar'V Exam i n ati o n• 2008
uctured Pc rlormnncc Ev a lua t lolJ (_OS P E)

Inter1,tive station 10
Mark.s: 05 Tln,e Allowe d: OS minutes

For Candid;,,te;

Task;
Carefully answ11.r to 01t.•mln , •• q v••t lc>l'l•1
1 . Man.. tor dC\"P 11\ou n,11 , nQ t•n th• t1 I· ,
1.s
2 . tiO\'i do yoo m•r tc,, dl' ri 1ngv11,111 ting"
05
3 . \Vhere you ,,rtt g,~~ an inc'1r.lon tor 10011,11 1 t""•i'I•" r~J1t7
01.
4. What is HemlOt.OfflV'
01
s. What ts tenStOO tree repair or " " 1 ,gull'\OI hemta ~ 1
1

'


Object i vely Structured Perfow~ncc Evaluatlon ( O~ l't:J

Interactive station 10
Marks: OS Tlme Atlowe.d: OS m inutes

For Examiner;

KEY:
01

ObSt.•rvt• lh~ '-I Udl"l'l l ,r en,, I

05

05

01

at neck and remo,11,q tn..• rema1n1no sac, 01

s. Mesh repair 01


ta JU "n
· • - ............. • ....... "'""'"""'utEv
Ob Pcrf°fCJ(Tcc nlu a t ,on (O S Pf. }
']e t:t lv cl y S t ru ct ur ed

In te ra c tiv e S ta tion 1 1
M. i r ks : 05
Tim o Al low ed : OS n11nu1c,

F or Ca nd id ate;
T as k:

r vo th llt c,v ht. -,1 11 hr>tor,,•11h,. ht en ttry •no f.lJtpta,n


C.:1refu lly o b ,-c
t1u11tt1,,n,1
th ei r u c~ b y ~·u , wt •r ln y eta 9'10 1n 11 ,'

-- ---------- -- •• .. ----- - - -- ----------

e ..
0 ...
I
C

...
.,' 0

"' .
o-
N tij

l
""'•• ••-.._.• ._,..._. ........... ,v,,~ "'-Y 4V

Objectively S truct ured Perforr e Evaluat ion (OSPE)


Static Sta :Tdn 9
Marks: OS Tln,c11.~1towed: OS minu..tcs
For Organizer:
Requirem ents:
l. Response·Sheet 2.Pen/ball point 3.Table 4>Chair /Stool

Top jc $pecifj<;ation: Orth opaedics


Pleaso cut along the dotted line nnd place the fol/owing photogr-nph on this station.
·---------------------------------------- ------------- -·

. -- - --- - --- - - - - - - - -- - - - - -- - - - - ...


Mark$: OS
---- - - - - - -- - - - - - - - - -- - - - -
Timo,A)lciw cd: Os minutes

For candidate;
35 y,ear, old mate w,ho suffered frem fra<>ture of right femur 4 months ago present'ed to
OPD with lnabllit,y to bear welgbt en. r.lght le.g. Clinleal exanifna'tlo.n rev.e.~ls tene!,er-nes.s
and excessi,;,e -mabili~y in -mrc•thlgl'i. x,;.ray ha_s been ,displayed.

Ja§ki
Carefully read t he scenario, examine the given ra!llograp.h and -answer the
followi'ng, questions:,
1. Des<>rlbe· the finding.~ an radtogr:apb. ot
2,. What e0'1)1pJlcatien· of f~_cture ha.s,'o,cG~rred? 01
3. Enumerate othe~•compllcatloos of:friicture. '02

MarkS: OS
for Examiner;
Key; 02 l
1 .. Non-union, perlostea.l elevatian, sctecosls,of fracture margins.
01
2. N'o:n-unlon.
0,2
3. Neurovascular~rn,a.,._11i{~ ~ ~u.~ ~mat..ualo11, 9§..t,eom eiitls.
FQr Organjze r:
Requirem ents: 102
1. Response Sheet 2.Pen/ bal l point 3.Table 4.Chair /Stool

Iopic Specification:
Please cut along t he dotted line and place the fol/owing photograph on this slat /on.
·------------------------------------------------------·

C:
0
.:;
,sg
I.I)

·-- --------------------- --- -------- --------------------·


Marks: OS Time Allow ed: OS minut es

Fo r Candidate :
20 year old male has present with 2 weeks hlstoiy of high grade fever and abdominal
pain. For the last two days t he abdominal pain has. aggravated and there is constipation.
On examinatton, patient Is toxic, febrile and dehydrated. His abdomen is distended anp
tender all over. His X-ray ehest·in eredi posture is shown:
_ ..,... ~]:ask! ..-... - •

earefully examine tjle given rad lograph and answer tile follo,wlng questions:
1. What are J he. ra,dlotogtcal findings? 01
2 . What 1.s your- diagnosis? 01
3. What Is .the. Initial m.an.ag.e(n~ll~of this, r,;a,tle-nt?, 02
01
4.· Whatls detlnitlve ,ueatment In this pa.tlenf?
.>
M.arks: .0 5
T ime '}l•~ w.eo :. u~ n J

For Examiner;
Ke)(;
1. G~s under d1apflra~m.
~"'• .0 1
2. Typhoj9 pe,rforation of terminal ileum.
3. ResuscJtation wlth IV ffuk:1s, N~, POley's, Aflfili5!t)tlt£. 02
4. Exploratol'Y, laparotomy,.1 r~air of, ~tfot~Uen 'e (¢. ei
- ....... a, ~A.dJn1nattons 2010
Objectively Structured PerW& nce Evaluation (OSPE)
Interactive Station 10
Marks: OS T•111e_A1Jowed: OS m inutes
For Organizer:
Requirements:
l. Resp_
onse Sheet 2.Pen/ball point 3.Table 4.Chair /Stool
Topic Specification: General Surgery & Trauma
Please cut along the dotted line and place the fol/owing photograph on thfs station.
·------------------------------------------------------

Marks: OS l'lme Allowed: 05 minutes


For Candidate:
Task;
C_arefully rea d t he scenario, observe th·e given pl:loto.graph/equipmept and
answer the following questions:
1:· (dentlfy tlils tubular ·equfpm·e )lt. oi
'
2. In which surglcal p1trceaure it is o·s-ed. ~.,.e ~ I 0,1
3. How yo·u manage ft post ola).el"atiyeJy . {-:r-~, ...~ 03

Marks: 05 Time Allowed: OS minutes


For Examiner:
Key:
1. T•Tube
2. CB.D explorat ion
iograrri, removal a-round 2 weeks if ·cholang_iogram is normal. ✓
3. T- tube cholang_
M ark:s: 05 Tfn,e Allo\vcd: 05 minutes
for Organi:ter: 104
Requirements:
1. Respoose Sheet 2.Pen/bl.lll•·Point 3..T-abfe 4.~hair /Stool
Topie Soecificatjon; Genera l.Surgery
Pleas'e ,ut along the dotted line and place, the following photogr;,ph on this station.
·-------------------------------------------------~----·

. --- - - - ----- - -- -- -.- - - -- - - - - -- - - - - - -- - --- -~-- - -•- -·------·--


M'&rks:,.0 5 Time AltoW~d: 05 minutes
Eor Candidat_@; .
'A 70 -rear,ora fern~~ .p.res~.nre~ ~lth·~rildval
. ·oo'set
. ot palntess·~aundi~. ,.,d_ _
Task:
Gare'.fully examine.t h e given ra9 iogr,~ph.a.n.il.a11.swe.f the (oil.owiii'g q!le~lon.s,(
i. f(lame,type '9f-111v~i ti~at)l'n ·(cijotang,oiram) whit.th l:ia§clfue'n p,-errp~mecl. '.01
:2. W,ha!:'~reJ:he flnclin'gs? ·OJ.
' 1
3:, What ar,.e tl\e;otl\er ·type.s. G.f,t0o_l,{n·wq9rami~l\'fc,H'<l):e mo.re frequ,entJy j)M(0/~( 101,
~ Wfii,t/c§titd ~ J h:e.M,h,i!l!rly~ g-~l)o!pg.y:·in,thls: ijatleii.t?· .01.
~ .

• ~

/>'.. W~tiar~ J:he co1rr1ltleatiori's 9f,mf.~~ -giogts_9t~m. fil


Marl(s: 05 Time Alld'we.~ : Q'S minl(t~.
For Examiner:
Key: . ..,
1. PTC. 01
2. Biliary dilatation with obstruction at the lower erit:l of ~ BD. i01
3. ERCP, MRCP 01
4. Cholangiocarcinoma, CA tread of panereas. Ol
5. Bile tea~ sepsis 0~
~ _J;,•

Annual Exa niln.itl,i~ ).010


Objectively Structured Perlorm4 ~Evalua.tion (OSPE)
Static Station 7
Tlma All~w;>d: OS.minute,,
M3rko: 05
for Organizer.:.
Requirement~:
1. Response Sheet 2.Pen/ball point 3. Table 4.Ch.llr /Stoo.l

Topjc Spec1ficatjon; Neurosurgical spin e


Plcti&o cut 111<>ng th" dotti!d lino :,nd place the fol/owing photograph Oft this st;atlon.
-------------------------------------- -----------------·

·--------------------------------------- ---j\llowcd:'
Marks: OS
------------ Tlmo 0~ mlnu~U
FQF Candidate;,
A 30 ,y!?ar old male preS'e ntlng. with ch'i'ontc bacl<ach·e.• l;l~"has ,histor,y l5t w~fgh,t 16ss.and
night sweats.• On clinical exa'rilir(ation! the patlent was this and emaciate<! with glbbus
formatioJ.l atth~ leV,el' !)f lowel' thor~i splne,
l'slsk;
.~ ffe.ful!Y i:ead the scenar~oi· examine the-alven·ra'iiiogra.ph and 11nsWer the
t'dJlow!rici qu~~iji>M: ~ .~
l,. What'are<t-he-~~<!~lo9,1$ 1 fli:\.'clJngf?. .li,S
~ , •Whn_t ~-you1 !>Job11bJe .dlagnosTs? 1.-s
'3. ijo)ll would .v,o-u t feat- this c»nditf~? 02

M.arks-: OS Time Allowed: OS minutes


For Examiner:
Key;
1. Wedge fracture of vertebral body. 1.5
2. Spinal tub~rculosls. 1.5
3. Anfituber.culous--1:J:le~apy witb ~ dr,ugs..-1lifampicil'l IN!,! ethamQutol aJ1d pyrazinamide
•••• , ... ,-·u1wv-,~1., . V-' IIIIIIU\\::~ .
Marks: OS
For Organizer; 106
Requirements:
1. Response Sheet 2. Pen/ball point 3.Table 4 .Chalr /Stool

Topic Specjflcation: Anaesth esia


Pl1>11se cur along the dotted /Inc 1tnd place t he fol/owing photcgr;,ph on this station.
·-------------------------------------------- ----------•

·------------------------------------------------------
Marks: 05 min_l(tes Time Allowed: 05
For Candidate~
Task;
Caref';'IIY examine the given 1n
. strument i.nd aos.w er the fpllowlng quest~o:ns:
t . ,fdentlfy the equlpn,'cnt, " o.s
·2 . Which d rug c.in you lnje\:t throtrgh It and whete. 1.S
3. \•lhk:h Is the most common ,col!)plicat(on after t'1i$, type_ot. ·anaesthesia a11d 119\Y call t!lis ,b.<!
preve~{ed. .i:.~
4', tlame,.tlJ)'.ee ~urgk:111 pro.cedur.es whk11citn bo oer.ro,med:uird~~ fills a~aes~sla. 1.s
Marks·; os Tl me.Allowe.(f: 05 nil,nUt•
For Examiner;
Key:
J... Lumb,ar Pori'cture. nll,~le. 0.$
2. lnj, Bupivacalne / lhj. Xylocaine. - Suparaonnold space.
1.S

i Hypote,nsJon --p;tload W
1tb I~ fl~id/ ·lnt: E~~e~,ihe-~
4'. Hcmo11b1d,ec1omy, lf!fulqal ~ rn!a.r,1?ait,Jf\/.RP(\t~
Annual Examln-!J:i,Q.q 2010
Objectively Structured Perfoi1"1Jf1;e Evaluation (OSPE)
"
Static Station 4
M<1rks: OS TlmQ Allowed: OS ml nu\..s
For Orqaniier:
~equirements:
1. Response Sheet :2.Pen/ball polnl 3.Table 4.Chair /Stool

Ioplc Specification; General Surgery


Please cut .;tong tho d~tlcd I/no ,,nd plar:c tho fol/owing photogn,ph on U1/s staHon.
·-------------------------------- ----------------------·

·---------------- --------------------------------------
M~rks: OS Time .Allowed: 05 minutes
For ~andidate; .
A. 60 year old ,nale lias prese'nted wfth multi1iie Rainless swellirig ·on the sGalP',.Th~y ,ar~
firm 111 co:nslstency and skin cannot- be plnc:hed over tllem.
Task-:
C~ref',!Uy read the,sc~na·rro, examin,e the given p,h'otogr-ap.h and an.s we.r t'1e
following questions:

t. Give your sJ)Ot ·dlatJD.Osis. o.i


~"v.:rat Is fjle f eatmen,t? 0:2
3. Wllat are tlteaoom))liitatlo»s:·af ~h1s •cori'<lltton? 02-
Marks: 05 Time Allowed_: 05 min~tes
For Examiner;
Key:
1. Sebaceous cysts. 01
2. Excision under local anaesthesla..
3. Infection wltt1 abscess forn1atlon, sebaceous horn.
(
Static St~ n 3
Maries: OS Time Allowed:. 0~ Minutes
For Organi;rer;
Requfrements:
1. Response Sheet 2.Pe,i/ball point 3.Table

Topic Soecificatjon; Gener al Surgery


Plo.,sC cut 1tlon9 the dotted linc' artd place th-e·followlng photograph on this statiofl.

·----------------------------- --------- ------------ ---- ~

.
•,- - - - -- - -- - -- -- - - - - - - - - ... -- - --..- -- ... - - - --·~--- -- - -- - --
':"• - ~ - ;-

M!Jrks: o.s Time ~llow"d{ os m.i nut~


For candidate:
A so yea" old Pa.in che,'iei:a)l!i_ cigaqjttif smo~e,r has p/es~_11.i:e(! with compl~ln}s;of ~l~r
'
ori right angle of mouth 'fo; the' last 6··months.:
Task;
Carefijlly read the se~n-4\rlo 'exarnln~ th~~ly~n pljoto:gtaph .t;niran~we_f1:he
following questions:

What ls·)i'our<djagoo~(!;.?
1.
Ji. ,Wh,'a..fii-,:e',tti.e l.?fijnsp;sJog'•f~ctors":t,:i,Jfils 1=ondi~I\?
- -~ Jl(hata~•the·fi:eatment options, fd'r' It?
Marks: OS --- - ____
~___...._
Time Allow:ed: OS minutes
For E-x aminer;
Key:
l. Squamous cell earcinoma 01
2. Tobacco, -aleohol, Pa,an, HPV, EBV,. 02
3. Surgery and external be·am radiotherapy 02
V U Jtt::i..-•l IVCI y .;,,41 U~LUI -.;iu ~ Cl I V I 1110 11 ~ ~ .:; VdlUdf.lUTI ( V:>t'l;;;J

Interactive SOQion 10
Marks: OS Time Allowed: OS mtnutes
Fo r Org<Jni~er:
Requirements:
1. Response Sheet 2.Pen/ball point 3.Table 4.Chair /Stool
Topic Specjfication: General Surgery
Please cut along the dotted lfnc and plat:c the fol/o w ing photograph on this station.

------------------------------------------- -- ----------·

. -- --- - -- -- - ---- - - - -.-- -- -- - - _,_-- -- - _._._- _,_,_-------- _.._- -


~

M.a rtcs; OS Tim.e .·Allo:wed: OS minutes


For candidate;
A•:25 year old man ln-.,olved In.a mot.I>r cyc;te··a9!aldenl, (levl!lqped·:seve1:e ~aln'al).d sw~III~.?
in tight thigh. He. has 15een bto.ught re·· sur9lc;_al emergency wltljln: half ar.i .hoyr c.after
SU'stalhl(!g>the 1.nJ\/!'Y,<_O)l exa1tti;ctqn his Pul~~ Is 116/mln aod ,Blood Rrt!ssure isJ~l_~O
mmHg arid meceJs :S&;m,. taceratl!'i'! w·o und 9n rigtit-uppet: thlgl). Hrs.,dlsJ:al s;,ulses a= llO.J:
~ a6le. His) <,r_l)yi9f)'(ilht (1:>lgh h.as l>'.een taken:· .. _,._ - - ., ,
Marks: 0 5 Tlm·e Allow·ed: OS minutes
For Examiner: I I
Key:
·1. There Is .a transverse fractu~e through the pro):(iiT-ial third ( Qiaphysis) of the right
femur, with 9r-OS!i displace.ment, ;:ingulation and overlap of. the fracture fragments. 01·
2. Suspected injury o,f Rigl.Jt Sup.erfici~I Femoral ;:irtery. 01
~- ABCDE. 01
4. External fixation :follqwed-b}:' vaswlar, F~pai_r; 02
--
Annual Examll)f',liA ns 2010
Objectiv.e./y Structured Perfof ~ ce Ev aluation (OSPE)
Static Station 9
M-,rk .s: OS Ttm-e Allowed: OS m inutes.
For Organizer;
Require·n,ents:
1. Response Sheet• 2-.Pen/ball point 3.Table 4.Chair /Stool

Jopjc Specification; Urology


Please cutato·ng tho dottt:d line·and place the follQwin9 photograph•on thls •sttJtion.
----------------------------------------------·---------·

- - - - - -~
- - - - - - - --- - -
Mlirks: OS
--·-... - - - - - ---- - - -- - - - - - - - -- -·- - - - - --- .. -·-- - -~
Time ~llovi(.cd: QS mlnut_e s
For candidar-e:
~O' Y:l!.iir:cild patient •fias presented with complaints of 1paln left loin.
Jask;-
eare'fu1Jr ·r4!ad ~ e ~senal'lo, -examll'!.!! the:;gi_v en radlci9'ra)>b 'and a n'swen~~
following questlons:
' -

1. Wha~ is.yo.u-r,_,d !,gQqsl~o)(I basis•of ~ata1fo1P.9 ica1.li~ing~? 02.


--
:2. Wh.a.,ti:a.f# •th!! ·treaffii-ent~i{flons?, 02
3. WhF<ffi -r epal.stQnes.ai'e cadloh~11t? 0-l
Merits: 05 lme A'flowitcl: OS mfn\ites
For Examiner:
Key:
1,. teft radi'c/J~cent u.~eter-ic stone. 02 9
2. URS w ffh dormia extractio11; .DJ stentln'Q; •Uc¢t:er-olithqt_~my
'3 . Uric acid•stones,,-Xantnint\ijstones,- - - - - - - - - - - , ,~
-··-- · ---- - - -------- --
H arles: 05
For o rganizer;
111 Time AJlow ed: 0 5 m i n vles

R e quireme nts:
I. Response S h eet 2.Pen/ball point 3.Table •I. Chair /St ool

Topic S peciffcation: Gen era l S urgery


Plca~u, cut along t he dott ed line a n d place th e Following p h otograph o n t his s tat i on.

·---------------------------- --------- --- --- -------- ---·

. - H.";ki:"'.'os - - - - - .... - - - - - - - - - - - - - - - - - - - - - - - - - - -TiineAiio\v"eci: Oi fflintitff- -


for candidate; ·
Task:
Carefully examine the given photograph and answer the following questions,
1. Is this sutufe naJural or sy n\_hetlc? o~
2. ts It absorbable o~ non-absort>llble? o~
3. \¥,ti.aj_'!_re Its ~~§1 _ _ 03,
Mar ks: 05. Time Allow.ed: 05 minutes
For Examiner;
Key: I
1. Sy~ etic f.lfi.t.,...I• 01~.,,
/,_
)I
,,.,
2. Non-absorbable 01 " V ·
3. Anchoring drains, scalP, sUtch lng,-Splenic~ilum.Jig_ation, occasionally for gut surg; ryi- if
Static st,~on 3
M3rks: OS Time Allow ed: 05 m inutes
For Organiier:
Requiren,ents:
1. Response Sheet 2 .Pen/ball point 3.Table 4.ch°{.1?"/Stool

Jopic Specification: Gener al Su rgery


Please cul <1/ong I/to dotted line and ploco tho following photogrnph on th is station.
----------- ----------------------- ----------------------
;.,..~ ,..
~:•·'"'-,-;-. ,,lf..,·'1
.
_..,..,..
.....
,
'..,-- •--~ .:-......•
-:. .. ... .,
-~"'--··
•.: .!-.i'Jli.
.. ~ • .. '
.·, .. . ---~-
.~
· .

....,... ..Iii .·- -. . ,,-


. .r·~,;'~.
,!· .;,,.,.__ra, j ._,.. :=;,. ·,•:·: ) ,,..,
~ . ~ . ~ . ... ' C
......~ '• .' ·~ . : 0
~\~ • • !.
' ... - f ,. -8
...,_·.• ; 1.. -..... . . ~
•• '1::-..
~~
...
..
."' . ~- ... ~... ,·,
,._.

·- ---------------- ------ ------------------ -------- -----


Mark$: 05 Tlme•Allowcd: OS minutes

For candidate;
A 32 year old shepherd has presentec;I with coml)laint of heaviness i n right
hypoch.ondrium for last 6 months. On examination, there fs mass in right hypochondrlum
which moves on respi ration and the upper limi t is not palpable. 0perative fin.dings are
Shown in pi~ure.

I i!Ski
Carefully re-ad th e scenario, observe th~ given photograph and ans,wer the
,following quest ions:
l. \'ihat Is ','our dl~nosJs? 01
;2·. ,W hat other orgal)'s can be lnvol\led by thJs dlse11se. 0~
3\ What Is medical treat tm!nt·of this dis.ease? 01,

4. W,ha~ ls PAlR? Ql ·

Mar1<s: OS l lf1MLA.!lifwe~:-OS ml nu
For Examiner;.
Keyi-
1: l'irdatt& l]ve~ d.ls~ase.
i. kun9;s, Spleen, Br-a'in, SubGutan'eou_s 'tiss.u~r
~- ;A_l~i!'n!:laz~le (:'400fu·g ·a ~)
4 . .Puti!>fi:l)fe, ~sp~tl<mn'i ,~~w~~
1
~lP-
St atic Stal 1an 4
Ma rks: 0 5 Tin, e Allowed: 05 mi nutes
For Organi;z;er:
Requirem ents:
1. Response Sheet 2, Pen/ball point 3.Table 4.Chalr /S\ool
Topic; Specification; General Surgery
Plense cut along t he dotted lin e a nd place the f ollo w ing photograph o n this stati on.
------- ------- ~--------- -------------------------------·

·-------------------------- ----------------------------·
Marks: OS Time Allowed: OS mi n utes
fort candidate:
35 year emaciate!:! male pres.e11ted with colicky abdomtnal pain, vemitirrg and dlsten.s.ion
for fast 3 da.ys. He has hfstort of r-ecurrent ·abdominal pain, vomiting and low grade fever
for last 6 months. He undet\Nent e·me(gency fa~arotQmy <)nd operative findlng_s are
shown.
Task;
Cai:.e(ul!v examine the given photo,gra"ph, reaj'I t~e scenario ·and answer t be
·fo.llowlng questions:
l.' What ls·you~ pre: ol)erat.111e ellntc,aJ di/ig nosl.s-1 01
·"' .,

·i: Wba.t •patliolo~lcf<1f' process Is lnv?l.v i.! l9JSmali ·ir:ifesti{ier · 'bi


,a. Mtrlt'(en ,t)vo qther c:ornmon o rgans ipv.cilve<l b~ ~fils disease. ·01
4?' What kpe_efflc'treatment will patlerit 'need on:c~ •reeove·recl,from,sorgery:? 02-
Time Allowed : OS minute"$
Marl<$::0 5

-For
Key:
Examiner :

1. fntestlnal obstruction. 01

2. Tuberculosis 01
3. Lungs, cerv ical lymph nodes. 01

4 . Antituberculous the.,@fill (!NH, Rifan,picln, Ethambutol, Pyrazinamide)


02
Static Stat1Wt 5
Marks: 05 Time Allowed : OS m i nutes
For Organj;z;er:
Requirements:
1. Response Sheet 2.Pen/ball point 3.Table 4.Chair /Stool

Topjc Specjfjcatjon: General Surgery


Plea.sc. cut t1l01tg the dotted lino and pfacfJ tho following photogrllph on th is st-ation.

------------- ----- -------------------------------------·

--------~----------------------- -- -------- -------------


Marks: 0 5 Time Allowed: OS minutes
For Candidate:
A 12 year old male presented with complaint of pain abdomen and anorexla (or last. 2
days. Pain started hi periumbilical region and shifted to right Iliac fossa, On examination,
there is tenderness and rebound tenderness ln right iliac rossa, Surgical .exploration was
c;arrled -out anp findings are sh.own.

Task;
Carefully read the scen ario, observe tl1,e given pliotograiih anct answer the
following questions:
1. What IS the operative ·,finding? OI
12. What surgical proce'dure has been performedi 81
3. Name .dlffer~n~ surgJ¢al inclsJon thro,ugh wh(~· tl,is proced(1re can be>performel;I
02
·,4_ What"comp(icatlon·aan .arise lf'lt ruptures·-p11~ ol?l!to\lvely·? O.~
~ .. .F, . ~ -- - --

Marks: 05 . Allowed:
Time . 05 minutes
.

f.or Examiner:
Keyi
1. Enlargement of appendix. (Mutocele appendixJ 0~
2. Emergency appendlcectomy. 01
3·. Gridiron, Lanz, Rutherford-Morrison, paramedian, mtlilline. 02
4. P5eudom,yxo,ma· ~o~on,ei 01
""'•• ••-.._.• ._,..._. ........... ,v,,~ "'-Y 4V

Objectively S truct ured Perfor;i1'{e Evaluat ion (OSPE)


Static Sta :l~n 9
Marks: OS Tln,c11.~1towed: OS minu..tcs
For Organizer:
Requirem ents:
l. Response·Sheet 2.Pen/ball point 3.Table 4>Chair /Stool

Top jc $pecifj<;ation: Orth opaedics


Pleaso cut along the dotted line nnd place the fol/owing photogr-nph on this station.
·---------------------------------------- ------------- -·

. -- - --- - --- - - - - - - - -- - - - - -- - - - - ...


Mark$: OS
---- - - - - - -- - - - - - - - - -- - - - -
Timo,A)lciw cd: Os minutes

For candidate;
35 y,ear, old mate w,ho suffered frem fra<>ture of right femur 4 months ago present'ed to
OPD with lnabllit,y to bear welgbt en. r.lght le.g. Clinleal exanifna'tlo.n rev.e.~ls tene!,er-nes.s
and excessi,;,e -m abili~y in -mrc•thlgl'i. x,;.ray ha_s been ,displayed.

Ja§ki
Carefully read t he scenario, examine the given ra!llograp.h and -answer the
followi'ng, questions:,
1. Des<>rlbe· the finding.~ an radtogr:apb. ot
2,. What e0'1)1pJlcatien· of f~_cture ha.s,'o,cG~rred? 01
3. Enumerate othe~•compllcatloos of:friicture. '02

MarkS: OS
for Examiner;
Key; 02 l
1.. Non-union, perlostea.l elevatian, sctecosls,of fracture margins.
01
2. N'o:n-unlon.
0,2
3. Neurovascular~rn,a.,._11i{~ ~ ~u.~ ~mat..ualo11, 9§..t,eom eiitls.
FQr Organjze r:
Requirem ents: 116
1. Response Sheet 2.Pen/ bal l point 3.Table 4.Chair /Stool

Iopic Specification:
Please cut along t he dotted line and place the fol/owing photograph on this slat /on.
·------------------------------------------------------·

C:
0
.:;
,sg
I.I)

·-- --------------------- --- -------- --------------------·


Marks: OS Time Allow ed: OS minut es

Fo r Candidate :
20 year old male has present with 2 weeks hlstoiy of high grade fever and abdominal
pain. For the last two days t he abdominal pain has. aggravated and there is constipation.
On examinatton, patient Is toxic, febrile and dehydrated. His abdomen is distended anp
tender all over. His X-ray ehest·in eredi posture is shown:
_ ..,... ~]:ask! ..-... - •

earefully examine tjle given rad lograph and answer tile follo,wlng questions:
1. What are J he. ra,dlotogtcal findings? 01
2 . What 1.s your- diagnosis? 01
3. What Is .the. Initial m.an.ag.e(n~ll~of this, r,;a,tle-nt?, 02
01
4.· Whatls detlnitlve ,ueatment In this pa.tlenf?
.>
M.arks: .0 5
T ime '}l•~ w.eo :. u~ n J

For Examiner;
Ke)(;
1. G~s under d1apflra~m.
~"'• .0 1
2. Typhoj9 pe,rforation of terminal ileum.
3. ResuscJtation wlth IV ffuk:1s, N~, POley's, Aflfili5!t)tlt£. 02
4. Explor~tOfY' laparotomy,.1 r~air of, ~tfot~Uen 'e(¢. ei
- ....... a, ~A.dJn1nattons 2010
Objectively Structured Perr,wance Evaluation (OSPE)
Interactive Station 10
Marks: OS T•111e_A1Jowed: OS m inutes
For Organizer:
Requirements:
l. Resp_
onse Sheet 2.Pen/ball point 3.Table 4.Chair /Stool
Topic Specification: General Surgery & Trauma
Please cut along the dotted line and place the fol/owing photograph on thfs station.
·------------------------------------------------------

Marks: OS l'lme Allowed: 05 minutes


For Candidate:
Task;
C_arefully rea d t he scenario, observe th·e given pl:loto.graph/equipmept and
answer the following questions:
1:· (dentlfy tlils tubular ·equfpm·e )lt. oi
'
2. In which surglcal p1trceaure it is o·s-ed. ~.,.e ~ I 0,1
3. How yo·u manage ft post ola).el"atiyeJy. {-:r-~, ...~ 03

Marks: 05 Time Allowed: OS minutes


For Examiner:
Key:
1. T•Tube
2. CB.D explorat ion
i ograrri, removal a-round 2 w eeks if ·c holang_iogram is normal. ✓
3. T- tube cholang_
SECTION ENDED
Some Examples of
Observed stations that you
may encounter in OSPE
For All those Who Are Asking What should we expect in observed stations of OSPE in Surgery, Here are a few scenarios

• Breaking Bad News For Carcinoma


• Consent for Anaesthesia
• Consent for Appendectomy
• Consent for Amputation
• Counselling that how a person will deal after amputation
• How a surgeon wears gloves before surgery
• Educating a Diabetic person that how he will care about his feet
• TB Informational care
• Consent for APR for Colorectal CA
• Consent for Lap Chole
• Consent for Prophylactic Mastectomy
• Consent for thyroidectomy in a singer who knows he can lose his voice in the process

You might also like